60
Question Paper 2 RBI Grade B 2019 Phase 1 General Awareness (Afternoon Shift) Prepare for RBI, SEBI & NABARD, UPSC & UGC NET Exam [email protected]; +91 9999466225 1. Where was the G7 summit 2019 held? Ans. Biarritz, France [Covered in the Course? Yes in August Spotlight] 2. Where is the headquarter of UNESCO? Ans. Paris, France 3. Manila is the capital of which country? Ans. Philippines 4. Who is the brand ambassador of MasterCard? Ans. MS Dhoni [Covered in the Course? Yes in October Spotlight] 5. Where is the Nakki Lake located in? Ans. Rajasthan 6. Which bank is being merged with Union Bank of India? Ans. Andhra Pradesh and Corporation Bank [Covered in the Course? Yes in September Spotlight] 7. Which state does the consumer affair and food distribution minister Ram Vilas Paswan belong to? Ans. Bihar 8. What is the percentage of gross NPA in banking sector in India, according to RBI’s Annual Report 2018-19? Ans. 9.1% [Covered in the Course? Yes in August Spotlight] 9. Bali Jatra festival is observed by which state? Ans. Odisha [Covered in the Course? Yes in October Spotlight] 10. With Which sport is Manju Rani related to? Ans. Boxing [Covered in the Course? Yes in October Spotlight] 11. 14th Nov is internationally celebrated as? Ans. World Diabetes Day 12. To which country does cricketer Ben stokes belong? Ans. England (DAILY) 13. For which sector does the RBI launched Central Information System for Banking Infrastructure (CISBI)? Ans. Cooperative banks [Covered in the Course? Yes in October Spotlight] 14. Who has authored The Daughter from a Wishing Tree book? Ans. Sudha Murthy [Covered in the Course? Yes in October Spotlight] 15. Where is the Kaiga national park located? Ans. Karnataka 16. Which state does the Kuchipudi dance form belong to? Ans. Andhra Pradesh 17. Where was the BRICS cultural meeting held? Ans. Curitiba, Brazil [Covered in the Course? Yes in October Spotlight] 18. With which sport is the Subroto Cup associated? Ans. Football [Covered in the Course? Yes in September Spotlight]

Question Paper 2 RBI Grade B 2019 Phase 1 General ... · 4. Who is the brand ambassador of MasterCard? Ans. MS Dhoni [Covered in the Course? – Yes in October Spotlight] 5. Where

  • Upload
    others

  • View
    3

  • Download
    0

Embed Size (px)

Citation preview

Page 1: Question Paper 2 RBI Grade B 2019 Phase 1 General ... · 4. Who is the brand ambassador of MasterCard? Ans. MS Dhoni [Covered in the Course? – Yes in October Spotlight] 5. Where

Question Paper 2

RBI Grade B 2019 Phase 1 – General Awareness (Afternoon Shift)

Prepare for RBI, SEBI & NABARD, UPSC & UGC NET Exam [email protected]; +91 9999466225

1. Where was the G7 summit 2019 held? Ans. Biarritz, France [Covered in the Course? – Yes in August Spotlight]

2. Where is the headquarter of UNESCO? Ans. Paris,

France 3. Manila is the capital of which country? Ans.

Philippines 4. Who is the brand ambassador of MasterCard?

Ans. MS Dhoni [Covered in the Course? – Yes in October Spotlight]

5. Where is the Nakki Lake located in? Ans. Rajasthan 6. Which bank is being merged with Union Bank of

India? Ans. Andhra Pradesh and Corporation Bank [Covered in the Course? – Yes in September Spotlight]

7. Which state does the consumer affair and food

distribution minister Ram Vilas Paswan belong to? Ans. Bihar

8. What is the percentage of gross NPA in banking

sector in India, according to RBI’s Annual Report 2018-19? Ans. 9.1% [Covered in the Course? – Yes in August Spotlight]

9. Bali Jatra festival is observed by which state?

Ans. Odisha [Covered in the Course? – Yes in October Spotlight]

10. With Which sport is Manju Rani related to? Ans. Boxing [Covered in the Course? – Yes in October Spotlight]

11. 14th Nov is internationally celebrated as? Ans. World Diabetes Day

12. To which country does cricketer Ben stokes

belong? Ans. England (DAILY)

13. For which sector does the RBI launched Central

Information System for Banking Infrastructure (CISBI)? Ans. Cooperative banks [Covered in the Course? – Yes in October Spotlight]

14. Who has authored The Daughter from a Wishing

Tree book? Ans. Sudha Murthy [Covered in the Course? – Yes in October Spotlight]

15. Where is the Kaiga national park located?

Ans. Karnataka 16. Which state does the Kuchipudi dance form

belong to? Ans. Andhra Pradesh

17. Where was the BRICS cultural meeting held?

Ans. Curitiba, Brazil [Covered in the Course? – Yes in October Spotlight]

18. With which sport is the Subroto Cup associated?

Ans. Football [Covered in the Course? – Yes in September Spotlight]

Page 2: Question Paper 2 RBI Grade B 2019 Phase 1 General ... · 4. Who is the brand ambassador of MasterCard? Ans. MS Dhoni [Covered in the Course? – Yes in October Spotlight] 5. Where

RBI Grade B 2019 Phase 1 – General Awareness (Afternoon Shift) 7

Prepare for RBI, SEBI & NABARD, UPSC & UGC NET Exam [email protected]; +91 9999466225

19. Which country has launched the Gaofen Satellite? Ans. China [Covered in the Course? – Yes in October Spotlight]

20. What is the capital of Sierra Leone?

Ans. Freetown 21. With which country does India conduct Vajra

Prahar military exercise? Ans. USA [Covered in the Course? – Yes in October Spotlight]

22. Which country has launched the Minuteman III

intercontinental satellite? Ans. USA [Covered in the Course? – Yes in October Spotlight]

23. Which country does the Nobel prize winner Peter

Handke belongs to? (Recently won Nobel prize in literature) Ans. Austria [Covered in the Course? – Yes in October Spotlight]

24. What is the reason for awarding William G Kaelin,

Gregg L Semenza and Peter J Ratcliff with the Nobel Prize in medicine 2019? Ans. For their discoveries of how cells sense and adapt to oxygen availability. [Covered in the Course? – Yes in October Spotlight]

25. Which state has hosted the India International

Cooperatives Trade Fair? Ans. New Delhi [Covered in the Course? – Yes in October Spotlight]

26. Bernadine Evaristo has been awarded with the Booker Prize 2019 for which book? Ans. Girl, Woman and Other [Covered in the Course? – Yes in October Spotlight]

27. Which country has abolished the Kafala System?

Ans. Qatar 28. Which ministry has launched the Dhruv- Pradhan

Mantri Innovative Learning Program? Ans. Ministry of HRD [Covered in the Course? – Yes in October Spotlight]

29. Which country will host the G7 Summit 2020?

Ans. USA [Covered in the Course? – Yes in August Spotlight]

30. Where is Kiaga Atomic Power Station located?

Ans. Karnataka 31. Which bank has launched the ‘FD Health’- fixed

deposit scheme? Ans. ICICI bank [Covered in the Course? – Yes in October Spotlight]

32. In which state does the Shinyuu Maitri exercise

between India and Japan held? Ans. West Bengal [Covered in the Course? – Yes in October Spotlight]

33. What does Article 22 elaborate?

Ans. Protection against arrest and detention in certain cases

34. Where was the women’s world boxing

championship held? Ans. Ulan Ude, Russia

Page 3: Question Paper 2 RBI Grade B 2019 Phase 1 General ... · 4. Who is the brand ambassador of MasterCard? Ans. MS Dhoni [Covered in the Course? – Yes in October Spotlight] 5. Where

RBI Grade B 2019 Phase 1 – General Awareness (Afternoon Shift) 8

Prepare for RBI, SEBI & NABARD, UPSC & UGC NET Exam [email protected]; +91 9999466225

[Covered in the Course? – Yes in October Spotlight]

35. How many languages does the new version of

BHIM 2.0 support? Ans. 3 (Haryanvi, Konkani and Bhojpuri)

36. To which country does cricketer Jennie Gunn

belong? Ans. England

37. What is the new minimum net worth threshold for

Bharat Bill Payment Operating Unit under the new on-tap authorization guidelines of RBI? Ans. Rs 100 crore [Covered in the Course? – Yes in October Spotlight]

38. Which organization has published the ‘South Asia

Economic Focus, Making (De) Centralization Works’? Ans. World Bank [Covered in the Course? – Yes in October Spotlight]

39. Merchant Discount Rates are exempted for

organization with an annual turnover of more than ___? Ans. Rs 50 crore [Covered in the Course? – Yes in October Spotlight]

40. Which payments bank has recently shut its

operation? Ans. Aditya Birla Idea Payments Bank [Covered in the Course? – Yes in July Spotlight]

41. With which bank has Home Credit India partnered

for co-lending? Ans. Karur Vyasa Bank

[Covered in the Course? – Yes in October Spotlight]

42. With which company has Airtel Payment Bank

partnered to launch the “Mosquito Disease Protection Policy”? Ans. HDFC Ergo [Covered in the Course? – Yes in September Spotlight]

43. With which bank has Bandhan Bank partnered to

launch the co-branded credit card? Ans. Standard Chartered Bank [Covered in the Course? – Yes in August Spotlight]

44. This is the ___ time that Tokyo is hosting summer

Olympics 2020? Ans. 2nd

45. Where is Green Park Stadium located?

Ans. Kanpur, Uttar Pradesh 46. Which state celebrates the Shirui Lily festival?

Ans. Manipur [Covered in the Course? – Yes in October Spotlight]

47. Cosmonaut Alexei Leonov has passed away.

Which country does he belong to? Ans. Russia [Covered in the Course? – Yes in October Spotlight]

48. Which is the second highest speaking language

after Hindi in India, according to Census 2011? Ans. Bengali – Census 2011 (ESI)

49. Where is the Padmaja Naidu Himalayan Zoological

Park located? Ans. West Bengal

Page 4: Question Paper 2 RBI Grade B 2019 Phase 1 General ... · 4. Who is the brand ambassador of MasterCard? Ans. MS Dhoni [Covered in the Course? – Yes in October Spotlight] 5. Where

RBI Grade B 2019 Phase 1 – General Awareness (Afternoon Shift) 9

Prepare for RBI, SEBI & NABARD, UPSC & UGC NET Exam [email protected]; +91 9999466225

50. Central Vigilance Commission has formed a committee, headed by T M Bhasin for examining bank frauds of over ___? Ans. RS 50 crore [Covered in the Course? – Yes in August Spotlight]

51. Which organization releases the State of World’s

Children Report 2019? Ans. UNICEF [Covered in the Course? Yes in October Spotlight] [Covered in the Course? – Yes in October Spotlight]

52. Which country has the lowest area among all UTs, according to the Census 2011? Ans. Lakshadweep [Covered in the Course? – Yes in Census 2011 (ESI)]

53. Abiy Ahmed Ali has been awarded Nobel Peace Prize 2019 for promoting peace with which nation? Ans. Eritrea [Covered in the Course? – Yes in October Spotlight]

54. Who has been awarded with the Lifetime

Achievement Award at India Sports Honours? Ans. Milkha Singh

55. Who has been awarded the best actor award,

along with Ayushman Khurana, at the 66th national Awards 2019? Ans. Vicky Kaushal [Covered in the Course? – Yes in August Spotlight]

56. Mahathir Mohamed is the Prime Minister of

which country? Ans. Malaysia

57. Who has become the first Indian women cricketer to complete 20 years in International Cricket?

Ans. Mithali Raj 58. Jagdeep Dhankar is the governor of which state?

Ans. West Bengal 59. Which country does King William and Queen

Maxima belong to? Ans. Netherlands [Covered in the Course? – Yes in October Spotlight]

60. With which country is the Frank Worrell Trophy related to apart from Australia? Ans. West Indies

61. Who has written the “How to Avoid a Climate

Disaster: The Solutions We Have and the Breakthroughs We Need” book?

Ans. Bill Gates [Covered in the Course? – Yes in October Spotlight]

62. Mark Vincent Hurd has passed away. He was the

Ex-CEO of which company? Ans. Oracle

63. Which state has topped the India Innovation Index

2019? Ans. Karnataka [Covered in the Course? – Yes in October Spotlight]

64. Where is the headquarters of Interpol located?

Ans. Lyon, France 65. Who has topped the Edelgive Hurun India

Philanthropy List 2019? Ans. Shiv Nadar [Covered in the Course? – Yes in October Spotlight]

Page 5: Question Paper 2 RBI Grade B 2019 Phase 1 General ... · 4. Who is the brand ambassador of MasterCard? Ans. MS Dhoni [Covered in the Course? – Yes in October Spotlight] 5. Where

RBI Grade B 2019 Phase 1 – General Awareness (Afternoon Shift) 10

Prepare for RBI, SEBI & NABARD, UPSC & UGC NET Exam [email protected]; +91 9999466225

66. Where has world’s first AI university been established? Ans. UAE [Covered in the Course? – Yes in October Spotlight]

67. To which country has India gifted a pair of Mi-24 helicopters? Ans. Afghanistan [Covered in the Course? – Yes in May Spotlight]

68. Where is Our Lady of glory Church located? Ans. Mumbai [Covered in the Course? – Yes in October Spotlight]

69. Where is the headquarters of International Boxing Association located? Ans. Lausanne, Switzerland

70. What is the theme of International Girl Child Day 2019? (Also asked in Morning shift) Ans. Girl force: Unscripted and unstoppable

71. Which company has for the first time crossed the Rs 9 lakh crore market capitalization limit? Ans. Reliance [Covered in the Course? – Yes in October Spotlight]

72. Which bank has launched the DigiSmart card? Ans. Standard Chartered Bank [Covered in the Course? – Yes in September Spotlight]

73. Which country has announced to leave OPEC by 2020? Ans. Ecuador [Covered in the Course? – Yes in October Spotlight]

74. Paektu Mountain is the highest mountain range of which country? Ans. North Korea

75. With which sport is Li Xuerui associated? Ans. Badminton [Covered in the Course? – Yes in October Spotlight]

76. Which company has planned to invest $1 billion in India to set up a local payment processing center? Ans. Master Card

77. Where is the headquarters of International Solar Alliance located? Ans. Gurugram, Haryana [Covered in the Course? – Yes in October July]

78. Who has been awarded with the Golden Shoe award 2019? Ans. Lionel Messi [Covered in the Course? – Yes in October Spotlight]

79. Which airline has become the world’s first airline to use Taxibot on A320 aircraft with passengers on-board? Ans. Air India [Covered in the Course? – Yes in October Spotlight]

80. State trading corporation of India operates under which ministry? Ans. Ministry of Commerce

Page 6: Question Paper 2 RBI Grade B 2019 Phase 1 General ... · 4. Who is the brand ambassador of MasterCard? Ans. MS Dhoni [Covered in the Course? – Yes in October Spotlight] 5. Where

RBI Grade B 2019 Phase 1 – English 11

Prepare for RBI, SEBI & NABARD, UPSC & UGC NET Exam [email protected]; +91 9999466225

1. Select the correct alternative from the given choices. Mideast University applies lowered academic cut-offs for athletes seeking admission. Hence, the admission office of the University requires an athletics office to enable them to judge athletic ability and offer suitable modification in admission standards. It can be inferred from the passage above that:

a) Students with athletic credentials generally fall short in academic standards

b) The university considers students with athletic achievements as better suited for admission than those with academic qualifications.

c) There is no coordination between the admissions office and the athletic office.

d) The university has very high academic admission standards. Ans: a

Explanation – If athletes are given modified academic admission standards, it follows that they do not generally meet the required standards. So (A) is the correct choice. (B) cannot be inferred from the passage. (C), if anything, contradicts the passage. (D) again, is not a definite inference.

2. Select the correct alternative from the given

choices Management Consultant It is generally true that Chinese restaurants that advertise their presence charge more for their services than Chinese restaurants that do not advertise. It is also true that each time advertising costs go down; more Chinese restaurants advertise their business and Chinese food becomes more expensive as a result. However, when advertising costs go up fewer Chinese restaurants would advertise their presence, and since they no longer have

the incentive to charge premium prices. Chinese food would be more affordable in consequence. In the management consultant’s argument, the two underlined portions play which of the following roles? Underlined 1- each time advertising costs go down; more Chinese restaurants advertise their business and Chinese food becomes more expensive as a result Underlined 2- since they no longer have the incentive to charge premium prices. Chinese food would be more affordable in consequence.

a) The first is a generalization that the

management consultant accepts as true, the second is presented as a corollary that follows from the truth of that generalization.

b) The first is a pattern of the cause- and – effect that the management consultant argues will be observed in the case at issue: the second acknowledges a circumstance in which that pattern would not hold.

c) The first demonstrates cause- and – effect that the management consultant accepts as true: the second is another example of the same relationship.

d) The first acknowledges a consideration that supports the main position that the management consultant defends, the second is the converse of that position. Ans: c Explanation- It is necessary to understand the management consultant’s main point: when advertising costs are affordable. Chinese restaurants (who advertise) charge higher prices on their menu and vice versa. The first underlined portion shows the cause- and – effect of falling advertising costs and

Page 7: Question Paper 2 RBI Grade B 2019 Phase 1 General ... · 4. Who is the brand ambassador of MasterCard? Ans. MS Dhoni [Covered in the Course? – Yes in October Spotlight] 5. Where

RBI Grade B 2019 Phase 1 – English 12

Prepare for RBI, SEBI & NABARD, UPSC & UGC NET Exam [email protected]; +91 9999466225

raising menu prices. The second underlined portion is simply its converse.

A. The second underlined portion is the converse of the first, not its corollary

B. The second underlined portion is not a unique circumstance.

C. Correct. The first underlined portion gives a general cause- and – effect relationships, the second demonstrates it again.

D. The first underlined portion gives a general cause- and – effect relationships, not a special consideration.

I.3-5) Read the passage carefully and answer the questions which follow. Faced with a streak of events and the necessity to make a choice, people may make one of three possible inductions: (1) that the streak is irrelevant, (2) that the streak will continue, or (3) that the streak, will stop. If people generally accepted the first of these Inductions, then when faced with a forced choice, they should predict the next event with a probability equal to its base rate. However, what is often observed is a bias toward one of the other two inductions even when the events are independent. The induction that the streak should continue was observed by Gilovich, Vallone, and Tversky in their study of the basketball phenomenon known as the hot hand. They found that basketball fans believed that a streak should be more likely to continue if a basketball player experiences a streak of hits than if that player had experienced a streak of misses. Yet Gilovich et al also showed that streaks of hits are no more likely than chance. The third induction is often known as the gambler's fallacy, a tendency to believe that a streak of events is likely to end. Laplace first wrote about this phenomenon, and its existence has been well documented.

Tversky and Kahneman's explanation for belief in the gambler's fallacy was that it is due to the representativeness heuristic, leading to a belief in a law of small numbers. In order for a sequence of events to be considered representative, people think that every segment of a random sequence should reflect the true proportion. Thus, a streak of one type of event must quickly end and be evened out by other events. Gilovich et al argued that belief in the hot hand is also due to belief in the law of small numbers. A belief that things should even out will be challenged by a long streak; therefore, basketball players may reconcile the apparently unusual streak and their belief in the law of small numbers by assuming that the events are dependent. Falk and Konold have pointed out that, in general, representativeness offers a convincing account of what participants do when judging random sequences but that its predictive power is weak. Similarly, Gigerenzer has pointed out that explaining the opposite phenomena with the same principle raises problems, yet this is what has been done with the gambler’s fallacy (that is the streak should stop) and the hot hand (that is the streak should continue). A step towards understanding how people use streak information would be to understand what distinguishes situations in which people tend to think a streak will continue from those in which they tend to think it will stop. The law of small numbers cannot explain this.

3. Which of the following is an instance of ‘Gambler’s fallacy’ at play?

a) A gambler winning by betting on a certain number, and in subsequent chances betting on the same number hoping to win.

Page 8: Question Paper 2 RBI Grade B 2019 Phase 1 General ... · 4. Who is the brand ambassador of MasterCard? Ans. MS Dhoni [Covered in the Course? – Yes in October Spotlight] 5. Where

RBI Grade B 2019 Phase 1 – English 13

Prepare for RBI, SEBI & NABARD, UPSC & UGC NET Exam [email protected]; +91 9999466225

b) A motorist being apprehensive about driving after being involved in an accident.

c) A person who after being rejected in love is unable to muster the courage to fall in love again.

d) A person pursuing his attempts to get through the ‘Kaun Banega Crorepati’ phone lines despite trying in vain for an hour. Ans: d Explanation- As per the passage a ‘gambler’s fallacy’ means that the streak will stop. Options (a), (b) and (c) are all contrary to gambler’s fallacy or display the reverse of gambler’s fallacy. Option (d) is the right answer, as it refers to the person’s belief that the failure will stop sooner or later.

4. As understood from the passage, what could be a reason for the basketball fan’s belief that a player is more likely to experience a streak of hits than misses?

a) Lack of analytical bent of mind of the basketball fans.

b) Media inputs to uphold the basketball fan’s belief.

c) Entrenched belief made stronger by selective observations.

d) Focus on enjoying rather than analysing the game. Ans: c Explanation – Option (a) can be easily ruled out as it is extremely farfetched. Options (d) can be ruled out as it contradicts itself as an answer since if the fans are more interested in enjoying themselves, they would not bother about, whether a player has a streak of hits or misses, they would not care about these incidental things. Option (b) can be ruled out as the role of media is farfetched one. Option (c) is apt as

an answer choice as the fans believe something and this gets strengthened when they witness something on the same line.

5. As understood from the passage, what can be

inferred from the last sentence of the passage? a) A streak is incomprehensible. b) Sample size alone does not help in explaining

people’s behaviour regarding streaks. c) People’s belief regarding streaks is contingent

on an adequate sample size. d) People’s analysis of gravitating between the

gambler’s fallacy and hot hand is contingent on their preconceived Ans: b Explanation – The last sentence says “The law of small numbers cannot explain this” option (b) is the apt answer. Option (a) can be ruled out as it is very general. Refer to the preceding sentence which says that people’s belief in a streak continuing or not is based on the situation. Hence sample size alone is insufficient. This makes choice (b) right and choice (c) incorrect. Option (d) is a distortion and hence can be ruled out.

I.6-8) Read the passage carefully and answer the questions which follow In the unusual properties of ‘mirror neurons’, a special category of nerve cell, scientists may have stumbled upon the brain mechanisms that give us the power to feel what others feel, to read others' intentions as though they were our own, and even to get deeply involved in the activity of others during a game of football or a dance performance. Yet more surprising, the properties of these mirror neurons suggest that human language began in gesture and mime, not in speech. And, finally, research links

Page 9: Question Paper 2 RBI Grade B 2019 Phase 1 General ... · 4. Who is the brand ambassador of MasterCard? Ans. MS Dhoni [Covered in the Course? – Yes in October Spotlight] 5. Where

RBI Grade B 2019 Phase 1 – English 14

Prepare for RBI, SEBI & NABARD, UPSC & UGC NET Exam [email protected]; +91 9999466225

defects in these nerve cells to autism and suggests novel methods for treatment.

The mirror-neuron story begins back in 1995, with a simple but surprising observation in the laboratory of Giacomo Rizzolatti at the University of Parma in Italy. Mr. Rizzolatti was measuring the electrical activity of brain cells in the premotor cortex of macaque monkeys. As its name suggests, this is a high-level area of the brain which plans and initiates body movements commands passed on from here result in electrical impulses going down your nerves to get your muscles moving. The Italian team found that particular nerve cells became active when a monkey reached out to pick up a peanut. That is what you might expect in a part of the brain involved in planning movement. Then the big surprise, when the monkey noticed one of the researchers reach out and pick up a peanut, exactly the same cells became active again. The brain cells fired whether the monkey itself did something or it saw another person or monkey do something with the same goal. Many more of these nerve cells that "mirrored" another animal's goal-directed movements were found. Before long, a much wider range of related mirror-neuron systems was found in human brains too. Why were these mirror neurons such a surprise? On a conventional view of how the brain works, sensations would arrive from the eyes, be analysed in the cortex and commands sent on to the muscle if action were required. The new experiments show that when we see someone carry out certain actions, the same parts of our brain are activated as if we were doing it ourselves. In essence: We don't need

to think and analyse, we know immediately what other people mean and feel by replicating what they do within the same areas of our own brains. As Mr. Rizzolatti puts it, “the fundamental mechanism that allows us a direct grasp of the mind of others is not conceptual reasoning but the direct simulation of the observed events through the mirror mechanism". That "direct grasp of the mind of others" is an ability that sets humans apart from almost all other animals, except the monkeys and apes, which have only the first rudiments of this skill. Understanding what others intend makes possible those unique human skills of debating, lying, cheating and manipulating, as well as limitation of others .Limitation is another skill that is almost uniquely human and permits learning to be passed on and culture to develop. Language, too, may owe its origins to the mirror neuron system. Obviously enough, any language has a first requirement that the person being communicated which understands the message of the communicator. Mirror neurons provide this first step directly. When a monkey performs an action with its hand, the mirror cells of another monkey will be watching will register the same action as if it were its own. Mirror neurons could have provided the starting point for the evolution of a language based on gestures which only later became associated with sounds. If our spoken language did come from gestures, then there should still be an overlap between the language areas of the brain and the motor areas. Not only does that turn out to be true

Page 10: Question Paper 2 RBI Grade B 2019 Phase 1 General ... · 4. Who is the brand ambassador of MasterCard? Ans. MS Dhoni [Covered in the Course? – Yes in October Spotlight] 5. Where

RBI Grade B 2019 Phase 1 – English 15

Prepare for RBI, SEBI & NABARD, UPSC & UGC NET Exam [email protected]; +91 9999466225

but scanners also show that one of the most important speech areas is active when we speak, when we gesture and when we see others gesture- just what would be predicted from a mirror- neuron origin of language. What happens if mirror neurons go wrong? We might expect that people would lose their ability to have a direct, intuitive feel for the mind of other people. That condition sounds very much like autism, which is characterized by defects in social interaction. Recently, Vilayanur Ramachandran and his team at the University of California in San Diego measured the activity of mirror- neuron areas in people by looking at a particular kind of brain wave, called the mu rhythm, which is suppressed when mirror nerves become active. Sure enough, the team found that mu wave was suppressed when people either moved their hands or watched others do so. But the mu rhythm of autistic people changed only when they move their own hands and not when they saw other people do the same thing. Autistic people indeed appear to have lost that direct link between watching and feeling that the mirror system may provide. Research is just beginning, but these findings may lead to new therapies.

6. All of the following information is true

regarding ‘mu rhythm’, EXCEPT: a) It gets suppressed when mirror nerves become

active. b) It is not suppressed when autistic people

gesture. c) It is suppressed when people watch others

gesture. d) It is not suppressed when autistic people watch

others move their hands. Ans: b

Explanation – Choice (a), (c) and (d) are true- mu rhythm – suppressed- mirror nerves become active – people – moved their hands- watched others do so – but choice (b) is not true – mu rhythm of autistic people changed only when they moved their own hands and not when they saw other people do the same thing.

7. All of the following link languages in the mirror

neuron system EXCEPT: a) There is no evidence of a link between the

language areas and motor areas of the brain. b) Gestures are used as sign language by the

hearing – impaired. c) Brain scanners reveal that a speech area is

active when we gesture. d) A mirror neuron system can provide an origin

for a language based on gestures that later turned to sound. Ans: a

Explanation – All the choices link language to the mirror neuron system save (1) - ‘ if our spoken language did come from gestures – an overlap between language areas of the brain and the motor areas – turn – brain scanners- speech areas are active when we speak- gesture- (para 6) - Choice (a) negates what is stated in the passage.

8. The ‘direct grasp of the mind of others” could

result in all of the following, EXCEPT? a) Understanding the intentions of the observed. b) Learning through imitation c) The development of language d) None of the above

Ans: d

Page 11: Question Paper 2 RBI Grade B 2019 Phase 1 General ... · 4. Who is the brand ambassador of MasterCard? Ans. MS Dhoni [Covered in the Course? – Yes in October Spotlight] 5. Where

RBI Grade B 2019 Phase 1 – English 16

Prepare for RBI, SEBI & NABARD, UPSC & UGC NET Exam [email protected]; +91 9999466225

Explanation – All the choices are supported by the passage- ‘direct grasp of the mind of others’ - understanding what others intend

I.9-11) Read the passage carefully and answer the questions which follow Leading B-schools are looking to overhaul the MBA syllabus. Nitin Nohria, the newly-appointed dean of Harvard Business School (HBS), is said to believe that the way management is taught in B-schools contributed to the recent financial crisis. Srikant Datar, another HBS professor, and two of his colleagues have authored a book, ‘Rethinking the MBA: Business education at a crossroads’, that echoes this theme. It also laments the fact that MBA students are taking less and less interest in their classes and spending more time on networking and attending recruiting events. ‘The management myth: Why the experts keep getting it wrong’, provides pointers to why students in MBA programmes may be switching off. The author, Matthew Stewart, thinks that management theories don’t add up to much, so the MBA course itself means little. Stewart’s background is interesting. After completing a doctorate in philosophy, he took up a job with a management consulting firm. With no knowledge of or background in management, he found himself advising CEOs of top companies at a billing rate half-a-million dollars a year. In his book, Stewart dissects some of the more famous management theories and finds them hollow. His assault on management’s claims to being a science rests on scrutiny of key theories in two areas, organizational behaviour and strategy, and works that focus on the creation of ‘excellence’ in companies.

As an example of how pretentious management theory can get, Stewart cites the Hawthorne effect uncovered by one of the famous names in organizational behaviour, Elton Mayo. A researcher was studying the effect of workplace illumination on worker productivity at a plant, Hawthorne Works. When the illumination was increased, productivity went up; when it was turned down, again it went up. The same thing happened with various other interventions. Mayo concluded that the results were not due to an external intervention but to the fact that the employees found they were getting attention. This is the foundation of much that has followed in the field since: about empowerment teamwork, etc. But Mayo's finding hardly qualifies as a dazzling Insight. He was merely pointing out that, as Stewart puts it, “If you are nice to other people, they will usually be nice to you.” This is a simple ethical construct. The danger in wrapping it up in management jargon is that students will see through the puffery and switch off. Some of Stewart's sharpest criticisms are reserved for the genre of management books typified by ‘In search excellence' authored by two ex-Mckinsey consultants, Tom Peters and Robert Waterman. The book purported to list the attributes of high-performing companies after evaluating 43 such companies. Two years later, half of the excellent 43 were in trouble. Five years later, almost all showed signs of decline. A great deal of what is taught in the harder areas at B-schools, such as accounting, finance, and quantitative sciences, is based on sound theory and finds wide applications in business. But Stewart is right in saying that most of the hard stuff at B-schools can be picked up fairly quickly. It is the soft part that constitutes

Page 12: Question Paper 2 RBI Grade B 2019 Phase 1 General ... · 4. Who is the brand ambassador of MasterCard? Ans. MS Dhoni [Covered in the Course? – Yes in October Spotlight] 5. Where

RBI Grade B 2019 Phase 1 – English 17

Prepare for RBI, SEBI & NABARD, UPSC & UGC NET Exam [email protected]; +91 9999466225

education and makes for a rounded manager. It is not easily imparted through courses, such as ethics or leadership, that are staple of B-schools. Soft skills, Stewart points out, are what the old liberal arts education or humanities was all about.

9. Mathew Stewart would most probably favour a) dumping the MBA program together b) an MBA that combines the requisite

quantitative skills with a liberal arts education c) an MBA program based on humanities. d) a compressed MBA program based on

quantitative skills and liberal arts. Ans: b

Explanation – Refer to last para choice (b) is the best answer. Option (a) is an extreme step not recommended by the author. Choice (c) focus on humanities alone. Choice (d) suggests compression of the entire program whereas the author would recommend compression of only the quantitative part.

10. Stewarts’ comments on the Mayo’s finding

from the research on work place illumination __________?

a) expresses his conviction that working conditions have a definite bearing on worker productivity.

b) Depict his thought that all management research is the sham.

c) reflects his belief that management theories are no more than commonplace truths.

d) is in line with his view ‘as you sow, so you reap’. Ans: c Explanation – Refer to the third para from the end. His comment ‘if you are nice .......... ethical construct’ shows choice c to be right

11. Which of the following statements is NOT true, according to the passage?

a) Stewart believes that students are not very interested in MBA programs.

b) Stewart does not believe that management is a science.

c) Stewart has a doctorate in Business Management.

d) Soft skills make a well- rounded manager. Ans: c Explanation – Choice (a) is true – refer to the third para ‘............. students in MBA programs may be switching off. Choice (c) is true - ‘His assault on management’s claims to being science - (Para 5). (Para 7), choice (d) is not true.

12. The following question has a paragraph from

which the last sentence has been deleted. From the given choices, choose the one that completes the paragraph in the most appropriate way. Laboratory tests on the rocky filaments found no evidence to suggest they were remnants of Earth-based organisms that contaminated the meteorites after they landed, Mr. Hoover said. He discovered the features after inspecting the freshly cleaned surfaces of three meteorites that are believed to be among the oldest in the solar system. He being an expert on life in extreme environments, has reported similar structures in meteorites several times before _______________________

a) Mr. Hoover is not the only researcher to claim the discovery to alien life inside meteorites.

b) The implications are that life is everywhere, and that life on Earth may have come from other planets.

c) So far, none has been confirmed as the ancient remains of alien life.

Page 13: Question Paper 2 RBI Grade B 2019 Phase 1 General ... · 4. Who is the brand ambassador of MasterCard? Ans. MS Dhoni [Covered in the Course? – Yes in October Spotlight] 5. Where

RBI Grade B 2019 Phase 1 – English 18

Prepare for RBI, SEBI & NABARD, UPSC & UGC NET Exam [email protected]; +91 9999466225

d) Too many scientists, Mr. Hoover’s work recalled the adage that extraordinary claims required extraordinary evidence. Ans: c

Explanation – The passage talks about some remnants that were not earth based organisms, which suggested they could below to aliens. Later it talks about how a scientist found similar structures several times. Hence, it is clear that though they were proved to be those of aliens. (a) continues but does not conclude. (b) talks about life on Earth from other plants, need to be supported further. (d) is irrelevant to be the context of the passage as it involves other scientists to (c) concludes by relating to the findings mentioned in the paragraph. (a) continues but does not conclude. (b) talks about life on Earth

13. The following question has a paragraph from

which the last sentence has been deleted. From the given choices, choose the one that completes the paragraph in the most appropriate way. The initial excitement generated by universal adult franchise gave way to cynicism. The penitential exercise of elected representatives seeking the people’s consent for re-election became a ritual. The initial hopes and dreams of the majority, the poor were soon shattered with minimal changes in their lot. The original enthusiasm when faced with an option of voting for different parties gradually fade with the realization that the choices provided no reasonable alternative _______________?

a) Nevertheless, political parties soon realized the importance of identifying dominant

sociocultural demographic pressures within constituencies.

b) Representation of people in the legislatures and the government did little for their empowerment or upliftment.

c) Telangana is a classic example of economic growth without political space empowerment for its people.

d) Gradually, the sheen wore off. Ans: b

Explanation - (a) continues the idea but does not conclude. (c) is an irrelevant example as it talks about economic growth. (d) redundant. (b) is apt as it refers to the failure of universal ad franchise.

14. The sentences are given in each of the

following questions when properly sequenced, form a coherent paragraph. Each sentence is labelled with a letter. From among the four choices given below each question, choose the most logical order of sentences that constructs a coherent paragraph

A. Renaissance explorers relied on charts of their time and since these were gravely inaccurate, even the most capable leaders were faced by longer voyage without landfalls than they bargained for.

B. Storms, shipwrecks, hostile inhabitants were hazards common to all seafaring, Even in European Waters, and the explorer prepared himself as best as he could to meet these mishaps.

C. Their most common reaction was baffled alarm at finding sea where they counted on finding land; or finding land instead of finding expected sea passages.

D. The real agonies suffered by Renaissance explores arose from their encounters with the unforeseen.

Page 14: Question Paper 2 RBI Grade B 2019 Phase 1 General ... · 4. Who is the brand ambassador of MasterCard? Ans. MS Dhoni [Covered in the Course? – Yes in October Spotlight] 5. Where

RBI Grade B 2019 Phase 1 – English 19

Prepare for RBI, SEBI & NABARD, UPSC & UGC NET Exam [email protected]; +91 9999466225

E. Almost always the unforeseen involved delay and its aftermath- food shortage and disease extra weeks on the open ocean or months imprisonment in ice.

a) (B), (D), (A), (C), (E) b) (D), (E), (B), (A), (C) c) (A), (C), (D), (E), (B) d) (D), (C), (A), (B), (E)

Ans: a Explanation – Sentence (b) introduces the idea of the passage by giving examples of the common hazards for which the explorers were prepared. (d) extends the idea by saying their real problem was with the unknown, (a) tells us why they could not predict certain problems. (c) supports (a) by giving an example. Finally (e) talks about the aftermaths of the unforeseen – bdace.

15. The sentences are given in each of the

following questions when properly sequenced, form a coherent paragraph. Each sentence is labelled with a letter. From among the four choices given below each question, choose the most logical order of sentences that constructs a coherent paragraph

A. The unity of the virtues implied the existence of a single absolute good.

B. He tried to show that any particular virtue when understood in depth, was not different from others.

C. For Socrates, the goal of life is to rise by mean of the intellect to a realization of this absolute good.

D. The Socratic dialectic usually sought graduated to arrive at an understanding of the essence of a particular moral virtue by examining propose definitions for completeness and consistency.

E. Socrates wanted to establish more than just a list of universal definitions.

a) (A), (B), (C), (E), (D) b) B), (C), (D), (E), (A) c) (D), (E), (B), (A), (C) d) (E), (A), (D), (C), (B)

Ans: c Explanation - (d) is a general statement and hence, starts the passage (e) follows (d) because it talks of ‘establish more than just list’. The ‘list’ is given in (d). In (b), he refers to Socrates, (a) introduces the idea of absolute good and concludes it – debac

16. The sentences are given in each of the

following questions when properly sequenced, form a coherent paragraph. Each sentence is labelled with a letter. From among the four choices given below each question, choose the most logical order of sentences that constructs a coherent paragraph

A. The best snake charmers have always lived in the East.

B. Through the centuries Indian snake charmer has been given a great deal of critical attention by visitors determined to be objective and ‘bring out the true facts’.

C. Some of the things they do are not yet wholly understood.

D. Even where snakes are not objects of worship or deification, their mystic influence on the human mind shows up in the many-face phenomenon of snake charming.

E. In Greece and throughout the Mediterranean oracular serpents were considered to inhabited by the spirits of dead heroes.

a) (A), (B), (D), (E), (C) b) (E), (D), (A), (C), (B) c) (E), (B), (C), (A), (D) d) (A), (B), (E), (D), (C)

Ans: b

Page 15: Question Paper 2 RBI Grade B 2019 Phase 1 General ... · 4. Who is the brand ambassador of MasterCard? Ans. MS Dhoni [Covered in the Course? – Yes in October Spotlight] 5. Where

RBI Grade B 2019 Phase 1 – English 20

Prepare for RBI, SEBI & NABARD, UPSC & UGC NET Exam [email protected]; +91 9999466225

Explanation - (e) introduces the subject of the passage, (d) supports the idea by saying that even in places where snakes are not worshipped, they indulge in snake – charming. (a) continue the idea of snake charming. In (c), ‘they’ refers to the snake charmers in (a), (b) concludes by saying of all snake charmers, Indian snake charmers were given a great desire of attention -

17. In each of the following questions a few

statements have been given. You have to arrange them and choose the correct arrangement from the given options.

a. It depends on certain external factors as well. b. An understanding of these concepts is crucial

in evaluating the agriculture policies of other agrarian countries such as India.

c. The aggregate measurement of support (AMS) is not just the budgetary expenditure of the government to provide subsidies to the agricultural products.

d. The calculation of quantum of market price support is based on the price gap between the ‘applied administered price’ identified in the domestic support measure and the ‘fixed external reference price’ multiplied by the quantity of eligible production.

a) bcda b) bdac c) cadb d) bcad e) cabd

Ans: c

18. In each of the following questions a few statements have been given. You have to arrange them and choose the correct arrangement from the given options.

a. Hopefully, with this development, the L&T management and Mindtree promoters will cease their war of words on this issue and leave the decision to Mindtree’s public shareholders, who are the only entities who should be taking a call on the merits of the officer

b. With Larsen & Turbo (L&T) announcing an open offer to acquire a 31 percent public shareholding in Mindtree, the melodrama over this takeover seems to giving way to a more reasoned response.

c. Mindtree, which was toying with a share buyback plan to undermine this bid, has now dropped this idea.

d. It has set up a committee of independent directors to evaluate the offer.

a) Bcda b) dacb c) cdba d) bcad e) cabd

Ans: a

19. In each of the following questions a few statements have been given. You have to arrange them and choose the correct arrangement from the given options.

a. The latest news doing the rounds is that banks have asked the Reserve Bank of India to defer the implementation of Ind AS.

b. The reason: the impact of Ind AS could be immense on their P&L account and balance sheet, which are already extremely weak.

c. It looks like banks in India can never be out of the news for an extended period of time.

d. In its role as the regulator, the RBI might accede to the request since we are already in the middle of March and there is no guidance yet from the central bank on critical concepts of Ind AS.

Page 16: Question Paper 2 RBI Grade B 2019 Phase 1 General ... · 4. Who is the brand ambassador of MasterCard? Ans. MS Dhoni [Covered in the Course? – Yes in October Spotlight] 5. Where

RBI Grade B 2019 Phase 1 – English 21

Prepare for RBI, SEBI & NABARD, UPSC & UGC NET Exam [email protected]; +91 9999466225

a) Bcda b) Bdac c) Cadb d) bcad e) cabd

Ans: e

20. A statement is given below followed by two assumptions, now based on the information, identify which assumption is implicit. Statement: The State government has decided to appoint four thousand primary school teachers during the next financial year. Assumptions:

I. There are enough schools in the state to accommodate four thousand additional primary school teachers.

II. The eligible candidates may not be interested to apply as the government may not finally appoint such a large number of primary school teachers.

a) Only assumption I is implicit b) Only assumption II is implicit c) Either I or II is implicit d) Neither I nor II is implicit e) Both I and II are implicit

Ans: a

Explanation -Such decisions as given in the statement are taken only after taking the existing vacancies into consideration. So, I implicit while II isn't.

21. A statement is given below followed by two

assumptions, now based on the information, identify which assumption is implicit. Statement: A warning in a train compartment - "To stop the train, pull chain. Penalty for improper use 500". Assumptions:

I. Some people misuse the alarm chain. II. On certain occasions, people may want to stop

a running train.

a) Only assumption I is implicit b) Only assumption II is implicit c) Either I or II is implicit d) Neither I nor II is implicit e) Both I and II are implicit

Ans: e

Explanation - Clearly, the penalty is imposed to prevent people from misusing the alarm chain. This means that some people misuse it. So, I is implicit. The alarm chain is provided to stop the running train in times of urgency. So, II is also implicit

22. A statement is given below followed by two

assumptions, now based on the information, identify which assumption is implicit. Statement: If it is easy to become an engineer, I don't want to be an engineer. Assumptions:

I. An individual aspires to be professional. II. One desires to achieve a thing which is hard to

earn.

a) Only assumption I is implicit b) Only assumption II is implicit c) Either I or II is implicit d) Neither I nor II is implicit e) Both I and II are implicit

Ans: b

Explanation: Clearly, nothing is mentioned about the professional nature of the job. So, I is not implicit. The statement hints that one rejects a thing that is easy to achieve. So, II is implicit.

Page 17: Question Paper 2 RBI Grade B 2019 Phase 1 General ... · 4. Who is the brand ambassador of MasterCard? Ans. MS Dhoni [Covered in the Course? – Yes in October Spotlight] 5. Where

RBI Grade B 2019 Phase 1 – English 22

Prepare for RBI, SEBI & NABARD, UPSC & UGC NET Exam [email protected]; +91 9999466225

23. A statement is given below followed by two assumptions, now based on the information, identify which assumption is implicit. Statement: "If I am not well you will have to go for the meeting." - A manager tells his subordinate. Assumptions:

I. It is not necessary that only manager level personnel attend the meeting.

II. If the manager is well, he would himself like to go for the meeting.

a) Only assumption I is implicit b) Only assumption II is implicit c) Either I or II is implicit d) Neither I nor II is implicit e) Both I and II are implicit

Ans: e Explanation - Clearly, the subordinate can attend the meeting as told by the Manager only when there is no inhibition. So, I is implicit. The subordinate is told to go only in case when the Manager is not well. This also shows the urgency to attend the meeting. So, II is also implicit.

24. In each of the question, four different ways of

presenting an idea are given. Choose the one that confirms most closely to standard English usage.

a) Although today, slavery has been abolished in

all of Europe and America, the people of Africa are still in a sense enslaved, for they face oppression every day, political, economic and social, that is a grim reminder of the enslavement of their people not long ago too.

b) Although today slavery has been abolished in all of Europe and America, the people of Africa are still in a sense enslaved, for they face

oppression every day, political, economic and social, that is a grim reminder of the enslavement of their people not long ago too.

c) Although today, slavery has been abolished in all of Europe and America, the people of Africa are, in a sense still enslaved, for they face oppression every day- political, economic and social – that is a grim reminder of the enslavement of their people not too long ago.

d) Although today slavery has been abolished in all of Europe and America, the people of Africa are, in a sense, still enslaved, for they face oppression every day, political, economic and social that are grim reminders of the enslavement not long ago too of their purpose. Ans: c Explanation – Only choice (C) is correct. The others have one or both of the two errors. (1) …. the people of Africa are in a sense, still enslaved …... The adverb ‘still’ qualifies verb ‘enslaved’ and so precedes it. The phrase ‘in a sense’ is an aside that is separated from the main sentence with commas on both sides. (2) The oppression is grim reminders of the enslavement of their people not too long ago. Too qualifies ‘long ago’ meaning not very long.

25. In each of the question, four different ways of

presenting an idea are given. Choose the one that confirms most closely to standard English usage.

a) Carl Sagan inspired the whole generation of a young scientist, especially in America, and his death by Cancer, in 1996, was a grievous loss to science and the whole world of reality-based thinking.

b) Carl Sagan inspired a whole generation of young scientists, especially in America, and his death from cancer in 1996 was a grievous loss

Page 18: Question Paper 2 RBI Grade B 2019 Phase 1 General ... · 4. Who is the brand ambassador of MasterCard? Ans. MS Dhoni [Covered in the Course? – Yes in October Spotlight] 5. Where

RBI Grade B 2019 Phase 1 – English 23

Prepare for RBI, SEBI & NABARD, UPSC & UGC NET Exam [email protected]; +91 9999466225

to science and the whole world of reality-based thinking.

c) Carl Sagan inspired a whole generation of a young scientist, especially in America, and his death from cancer, in 1996, was a grievous loss to science and the whole world of reality-based thinking.

d) Carl Sagan inspired a whole generation of young scientists, especially in America and his death by cancer in 1996 was a grievous loss to science and the whole world of reality-based thinking. Ans: b

Explanation – While it can be a ‘generation’ or ‘generations’, the noun scientist has to be in plural. Further death is ‘from cancer’ not ‘by’ - this rules out choice 'd' Only choice B is apt.

26. In the question, a statement is given below

followed by two conclusions. Based on your knowledge, identify the conclusions which follow? Statement: Start to think of traveling by train for a holiday. A train journey can give one a better view of places on the way which an air journey cannot give. Conclusions:

I. While going on a holiday, people want to enjoy the view of the places on the way.

II. People should not travel by air when they are going on holiday.

a) if only conclusion I follows. b) if only conclusion II follows. c) if either I nor II follows. d) if neither I nor II follows. e) if both I and II follow

Ans: d

Explanation - I is an assumption. II is a suggestion but not a conclusion. Hence, neither I nor II follows.

27. In the question, a statement is given below

followed by two conclusions. Based on your knowledge, identify the conclusions which follow? Statement: Vegetable prices are soaring in the market. Conclusions:

I. Vegetables are becoming a rare commodity. II. People cannot eat vegetables.

a) if only conclusion I follows. b) if only conclusion II follows. c) if either I nor II follows. d) if neither I nor II follows. e) if both I and II follow

Ans: d Explanation - The availability of vegetables is not mentioned in the given statement. So, I does not follow. Also, II is not directly related to the statement and so it also does not follow.

28. In the question, a statement is given below

followed by two conclusions. Based on your knowledge, identify the conclusions which follow? Statement: All the organized persons find time for rest. Sunitha, in spite of her very busy schedule, finds time for rest. Conclusions:

I. Sunita is an organized person. II. Sunita is an industrious person.

a) if only conclusion I follows. b) if only conclusion II follows. c) if either I nor II follows. d) if neither I nor II follows.

Page 19: Question Paper 2 RBI Grade B 2019 Phase 1 General ... · 4. Who is the brand ambassador of MasterCard? Ans. MS Dhoni [Covered in the Course? – Yes in October Spotlight] 5. Where

RBI Grade B 2019 Phase 1 – English 24

Prepare for RBI, SEBI & NABARD, UPSC & UGC NET Exam [email protected]; +91 9999466225

e) if both I and II follow Ans: a

29. In the question, a statement is given below

followed by two conclusions. Based on your knowledge, identify the conclusions which follow? Statement: Death keeps no calendar. Conclusions:

I. Man must die one day. II. Death can come at any time.

a) if only conclusion I follows. b) if only conclusion II follows. c) if either I nor II follows. d) if neither I nor II follows. e) if both I and II follow

Ans: e

30. In the question, a statement is given below followed by two conclusions. Based on your knowledge, identify the conclusions which follow?

Statements: The Government run company had asked its employees to declare their income and assets but it has been strongly resisted by employees union and no employee is going to declare his income. Conclusions:

I. The employees of this company do not seem to have any additional undisclosed income besides their salary.

II. The employee’s union wants all senior officers to declare their income first.

a) if only conclusion I follows. b) if only conclusion II follows. c) if either I nor II follows. d) if neither I nor II follows. e) if both I and II follow

Ans: d Explanation - Nothing about the details of the employees' income or the cause of their refusal to declare their income and assets, can be deduced from the given statement. So, neither I nor II follows.

Page 20: Question Paper 2 RBI Grade B 2019 Phase 1 General ... · 4. Who is the brand ambassador of MasterCard? Ans. MS Dhoni [Covered in the Course? – Yes in October Spotlight] 5. Where

RBI Grade B 2019 Phase 1 – Reasoning 25

Prepare for RBI, SEBI & NABARD, UPSC & UGC NET Exam [email protected]; +91 9999466225

I.1) Answer the questions based on the following instructions. Mark 1: If the Question can be answered by using statement A but not by using statement B alone. Mark 2: If the question can be answered by using statement B alone but not by using statement A alone. Mark 3: If the question can be answered by using either of the statement alone. Mark 4: If the question can be answered by using both the statements together but not by either of the statements alone.

1. In a class of 125 students if 20 students failed

in all three subjects – Physics, Chemistry, and Mathematics – then how many students passed in only Physics?

I. 65 students passed in Mathematics and 105

students passed in Chemistry. II. 65 students passed in Chemistry as well as

Mathematics, 60 students passed in Physics as well as Chemistry.

a) Mark 1 is correct b) Mark 2 is correct c) Mark 3 is correct d) Mark 4 is correct.

Ans: a

Explanation: A alone, As C =105 and n =20 C + N = 125 A = e = b = 0 A alone is sufficient From B alone we cannot find out the values of ‘b’ as it gives partial information

I.2-3) Answer the questions based on the following instructions.

Mark 1: If the Question can be answered by using one of the statements alone but not by the other. Mark 2: If the question can be answered by using either statement alone. Mark 3: If the question can be answered by using both statements together, but cannot be answered by using either statement alone. Mark 4: If the question cannot be answered even by using both the statements together.

2. Only if Shyam eats no more than 3 sweets, then

Ram will eat no more than three sweets, and Seeta will eat at least three sweets. How many sweets did Shyam and Seeta together eat, given that everyone ate a different number of sweets and each of them at least one sweet?

I. Seeta ate three sweets and Ram ate two

sweets. II. Ram ate one sweet and Seeta ate four sweets.

a) Mark 1 is correct b) Mark 2 is correct c) Mark 3 is correct d) Mark 4 is correct.

Ans: a

Explanation – The given statement is of the form of: Only if P then Q, The implications are: (1) q= p (2) -p = -q Given that Seeta had 3 sweets and Ram had 2 sweets that are q is true which means p is true that Shyam eats at most 3 sweets. But we know that all of them had a different number of sweets. Shyam had one sweet. The given question can be answered with the statement I alone.

Page 21: Question Paper 2 RBI Grade B 2019 Phase 1 General ... · 4. Who is the brand ambassador of MasterCard? Ans. MS Dhoni [Covered in the Course? – Yes in October Spotlight] 5. Where

RBI Grade B 2019 Phase 1 – Reasoning 26

Prepare for RBI, SEBI & NABARD, UPSC & UGC NET Exam [email protected]; +91 9999466225

As per statement (II), q is true which means Shyam is at most three sweets but since Ram had one sweet. Shyam can have either two or three sweets hence the question cannot be answered.

3. How many Tuesdays are there in the year?

I. The year started with a Monday.

II. The year ended with a Sunday.

a) Mark 1 is correct b) Mark 2 is correct c) Mark 3 is correct d) Mark 4 is correct.

Ans: a

Explanation – From statement I alone we cannot determine the number of Tuesdays in the year because if the year is a non-leap year there will be 52 Tuesdays. If the year is a leap year there will be 53 Tuesdays. From Statement II the number of Tuesdays in that year is exactly 52. Hence, II alone is sufficient

I.4-7) Study the given information carefully and the answer the following question below. A certain number of people sitting in a South facing row. Only some people arrangements are known. Only one person sits between I and M.L sits fourth to the left of I. Five person sits between L and V. Four people sit between V and Q. Not more than six people sits to the right of V. Number of person sits between I and Q is as same as between number of person sits between M and T. M sits right of T. Three people sit between T and W. W sits adjacent to L.

4. If B sits Exactly between V and I, then what is the position of L with respect to B?

a) Second to the left b) Fifth to the left c) Fourth to the right d) Fifth to the right e) None of these

Ans: b

5. Who sits third to the right of W? a) None of them sits third to right of W b) M c) I d) L e) T

Ans: b

6. How many persons are sitting in a row? a) 16 b) 20 c) 18 d) 17 e) None of these

Ans: d

7. Four of the five among the following are similar in such a way to form a group, which one of the following doesn’t belongs to the group?

a) T b) M c) L d) W e) V

Ans: a

I.8-11) Study the given information carefully and the answer the following question below Eight friends – Sundar, Satya, Mark, Cook, Putin, Obama, Trump and Nitish are sitting around the circular table facing centre, but not necessarily in the same order. Putin and

Page 22: Question Paper 2 RBI Grade B 2019 Phase 1 General ... · 4. Who is the brand ambassador of MasterCard? Ans. MS Dhoni [Covered in the Course? – Yes in October Spotlight] 5. Where

RBI Grade B 2019 Phase 1 – Reasoning 27

Prepare for RBI, SEBI & NABARD, UPSC & UGC NET Exam [email protected]; +91 9999466225

Obama are not immediate neighbours. Only two persons sit between Sundar and Trump. Obama is not an immediate neighbour of Trump and Cook. Putin is not an immediate neighbour of Mark and Trump. Sundar sits second to the left of Cook. Nitish is not an immediate neighbour of Putin. Only three persons sit between Mark and Obama. Satya sits not opposite of Cook. Trump is not an immediate neighbour of Putin.

8. What is the position of Cook with respect to Putin?

a) Third to the right b) Second to the right c) Immediate right d) Fourth to the right e) None of these

Ans: c

9. How many persons sit between the one who sits the second to the left of Cook and Mark, when counted from left of Mark?

a) One b) Two c) Three d) No one e) None of these

Ans: b

10. Which of the following statement is correct? a) Only one person sits between Nitish and the

one who sits to the immediate left of Sundar b) Cook sits to the immediate left of Trump. c) Only three persons sit between Satya and

Sundar d) Sundar and Mark is an immediate neighbour e) None of these

Ans: d

11. Who is sitting diagonally opposite to Putin?

a) Obama b) Trump c) Nitish d) Satya

Ans: b

I.12-14) Answer the questions on the basis of information given below. Archana, Bhargavi, Chandrika, Divya, Eshwari and Fathima are six faculty members in a college. Each faculty member is a professor in exactly one subject and an Associate Professor in exactly one other subject. Bhargavi is a professor in physics, for which there are three associate professors. Eshwari is a faculty member in Physics and Biology. Eshwari and Bhargavi are faculty members in the same subject. Archana and Chandrika are faculty members in Chemistry. Archana is an associate professor in Mathematics, for which Divya and Chandrika are professors. For commerce, there is exactly one professor.

12. Who is the Associate Professor in Commerce? a) Divya b) Bhargavi c) Fathima d) None of these

Ans: d

Explanation – We get the information in question and formulate the information Bhargavi is the head professor in Physics. Eshwari and Bhargavi are faculty in the same subject. Eshwari is a faculty member in Physics and Biology. Eshwari and Bhargavi -> Work in the same subject Eshwari -> Physics and Biology

Page 23: Question Paper 2 RBI Grade B 2019 Phase 1 General ... · 4. Who is the brand ambassador of MasterCard? Ans. MS Dhoni [Covered in the Course? – Yes in October Spotlight] 5. Where

RBI Grade B 2019 Phase 1 – Reasoning 28

Prepare for RBI, SEBI & NABARD, UPSC & UGC NET Exam [email protected]; +91 9999466225

Archana and Chandrika are involved in Chemistry

Head Professor

Associate Professor

Physics Bhargavi Eshwari, Divya, Fathima

Biology Eshwari Bhargavi Chemistry Archana Chandrika

Mathematics Divya, Chandrika

Archana

Commerce Fathima No one

Associate Professor in Commerce is Fathima

13. Fathima is a Professor in a) Mathematics b) Biology c) Physics d) Commerce

Ans: d

Explanation – We get the information in question and formulate the information Bhargavi is the head professor in Physics. Eshwari and Bhargavi are faculty in same subject. Eshwari is faculty member in Physics and Biology. Eshwari and Bhargavi -> Work in same subject Eshwari -> Physics and Biology Archana and Chandrika are involved in Chemistry

Head Professor

Associate Professor

Physics Bhargavi Eshwari, Divya, Fathima

Biology Eshwari Bhargavi Chemistry Archana Chandrika

Mathematics Divya, Chandrika

Archana

Commerce Fathima/No one

No one/ Fathima

14. Who among the following is/are Associate

Professor in Biology? I. Bhargavi

II. Eshwari III. Divya IV. Chandrika

a) Only II and III b) Only I c) Only I and II d) Only II, III and IV

Ans: b

Explanation - We get the information in question and formulate the information Bhargavi is the head professor in Physics. Eshwari and Bhargavi are faculty in the same subject. Eshwari is a faculty member in Physics and Biology. Eshwari and Bhargavi -> Work in the same subject Eshwari -> Physics and Biology Archana and Chandrika are involved in Chemistry

Page 24: Question Paper 2 RBI Grade B 2019 Phase 1 General ... · 4. Who is the brand ambassador of MasterCard? Ans. MS Dhoni [Covered in the Course? – Yes in October Spotlight] 5. Where

RBI Grade B 2019 Phase 1 – Reasoning 29

Prepare for RBI, SEBI & NABARD, UPSC & UGC NET Exam [email protected]; +91 9999466225

Head Professor

Associate Professor

Physics Bhargavi Eshwari, Divya, Fathima

Biology Eshwari Bhargavi Chemistry Archana Chandrika

Mathematics Divya, Chandrika

Archana

Commerce Fathima/ No one

No one/ Fathima

I.15-17) Answer the questions on the basis of information given below. Seeing the huge popularity of the Indian Premier League (IPL) tournament, the All India Football Federation started an Indian Football League (IFL) tournament, to give impetus to the sport in the country. Eight clubs representing eight cities of the country participate in the inaugural tournament, which comprises three stages. The first stage, called the group stage, is a double round robin stage that is a stage in which every team plays exactly two matches against each of the other teams. If any match in this stage ends as a draw, both teams get one point each. Otherwise, the winning teams three points and losing team gets no points. If at the end of the group, two or more teams end up with same number of points. Goal differences will be applied to determine their placings. Assume that no two teams end up with same goal difference. At the end of this stage, the top four teams, in terms of the points scored would advance to the third stage that is finals. The winner of the finals is the champion of the tournament. It is ensured that there are no draws in the second and third stages.

15. It is known that ‘Martha Stallions’, a Pune

based club, made it to the semi- finals. The

minimum possible points obtained by ‘Martha Stallions’ at the end of group stage are:

a) 8 b) 10 c) 11 d) 12

Ans: a

Explanation- For a team to advance to the semi-finals with least number of points with the least number of points, the top three teams should win the maximum number of matches while other five teams should draw all matches among themselves. One among these five teams would advance to semi-finals. Minimum number of matches between the bottom five teams = 5*4 Minimum number of total points of the bottom five teams = 20*2 = 40 Minimum points of 4th ranked team = 8*1 = 8 points

16. The number of points scored by a team which

does not advance to the semi-finals can be at most

a) 28 b) 26 c) 30 d) 32

Ans: c

Explanation – If five teams equally score the maximum points, one of these five teams could be eliminated. This can happen if last three teams only score points among themselves. Maximum points available for the top 5 teams = (56-6)*3= 150 Each of the five teams can score 30 points and one of them can be eliminated.

Page 25: Question Paper 2 RBI Grade B 2019 Phase 1 General ... · 4. Who is the brand ambassador of MasterCard? Ans. MS Dhoni [Covered in the Course? – Yes in October Spotlight] 5. Where

RBI Grade B 2019 Phase 1 – Reasoning 30

Prepare for RBI, SEBI & NABARD, UPSC & UGC NET Exam [email protected]; +91 9999466225

17. At least how many points should be scored by a team to finish sixth in group stage?

a) 4 b) 5 c) 6 d) 7

Ans: c

Explanation – The team which finished sixth can have the least points when the last three teams only scored points in matches among themselves and all matches among these teams ended in draws. Each team can score four points and the team with the best goal difference would be placed sixth.

I.18-22) Study the information carefully and answer the questions given below. Five geeks (enthusiasts) entered a comic book character costume contest held during comic con festival. The contestants dressed up and were given two awards – one for being best in a particular (that is strongest, smartest, most efficient, scariest and powerful) and one for being ranked (that is 1st, 2nd, 3rd, 4th and 5th). Determine who wore what costume and what two awards they received.

I. The winner of the most efficient character’s costume was ranked just above Sasha’s character’s costume (which wasn’t the superman)

II. The winner of the strongest category (which wasn’t the Wonder Woman) was not worn by Billy.

III. The joker’s costume was ranked just above Sacha’s costume (which wasn’t the strongest costume winner)

IV. The Wonder Woman costume ranked just above the scariest costume and just below Vladimir's costume.

V. The Batman costume was placed just higher than the winner of the strongest costume and just lower than Catherine’s costume.

VI. The Lex Luthor costume ranked just above Jelena’s (which wasn’t the superman) and just below the smartest.

18. Who among the following ranked fifth? a) Billy b) Jelena c) Vladimir d) Cannot be determined

Ans: a

Explanation – We follow the instructions and we get the following information

Person Costume Award

Catherine Joker Most Efficient

Sasha Batman Smartest Vladimir Lex Luthor Strongest Jelena Wonder

Woman Most

Powerful Billy Super Man Scariest

19. Who among the following got the award of the

scariest costume? a) Billy b) Catherine c) Sacha d) Vladimir

Ans: a

Page 26: Question Paper 2 RBI Grade B 2019 Phase 1 General ... · 4. Who is the brand ambassador of MasterCard? Ans. MS Dhoni [Covered in the Course? – Yes in October Spotlight] 5. Where

RBI Grade B 2019 Phase 1 – Reasoning 31

Prepare for RBI, SEBI & NABARD, UPSC & UGC NET Exam [email protected]; +91 9999466225

Person Costume Award Catherine Joker Most

Efficient Sasha Batman Smartest

Vladimir Lex Luthor Strongest Jelena Wonder

Woman Most

Powerful Billy Super Man Scariest

20. Which of the following is the costume of

Sacha? a) Joker b) Batman c) Lex Luthor d) Wonder Woman

Ans: b

Person Costume Award Catherine Joker Most

Efficient Sasha Batman Smartest

Vladimir Lex Luthor Strongest Jelena Wonder

Woman Most

Powerful Billy Super Man Scariest

21. Which of the following shows the correct order

of ranks 1st, 2nd, 3rd, 4th and 5th, respectively?

a) Billy, Jelena, Vladimir, Sacha, Catherine b) Vladimir, Sacha, Catherine, Billy, Jelena c) Catherine, Sacha, Vladimir, Jelena, Billy d) Vladimir, Billy, Catherine, Sacha, Jelena

Ans: c

Person Costume Award Catherine Joker Most

Efficient Sasha Batman Smartest

Vladimir Lex Luthor Strongest Jelena Wonder

Woman Most

Powerful Billy Super Man Scariest

22. Which of the following are not correctly

matched? Costume Category I. Joker Most Efficient

II. Batman Smartest III. Lex Luthor Strongest

IV. Wonder Woman Scariest

V. Superman Powerful

a) Only IV and V b) Only III and IV c) Only II and III d) Only I and II

Ans: c

Person Costume Award Catherine Joker Most

Efficient Sasha Batman Smartest

Vladimir Lex Luthor Strongest Jelena Wonder

Woman Most

Powerful Billy Super Man Scariest

I.23-24) Answer the questions on the basis of information given below

I. Six persons- A, B, C, D, E, and F – are sitting in

two rows of three persons each, such that each person in a row faces exactly one person in the other row.

Page 27: Question Paper 2 RBI Grade B 2019 Phase 1 General ... · 4. Who is the brand ambassador of MasterCard? Ans. MS Dhoni [Covered in the Course? – Yes in October Spotlight] 5. Where

RBI Grade B 2019 Phase 1 – Reasoning 32

Prepare for RBI, SEBI & NABARD, UPSC & UGC NET Exam [email protected]; +91 9999466225

II. F is sitting in the middle of a row. III. A and B are sitting in the same row but not next

to each other IV. D is sitting at the extreme left of one of the

rows and C is adjacent to him. V. E and B are not opposite each other.

23. Who is sitting opposite F? a) A b) B c) C d) D

Ans: c

Explanation – From (I) six persons are in two rows of three persons each as shown below From (IV) D is at the extreme left of one of the rows X and C is at the middle of row X From (II), F must be sitting in the other row and A and B are sitting in the same row as F. Hence, E must be in the same row as C. From (IV), A is sitting opposite E. D is sitting opposite B The final arrangement is as follows: E C D A F B C is opposite F

24. Which of the following pairs of persons are at

the ends of two different rows but not opposite each other?

a) A and B b) B and D c) A and E d) A and D

Ans: d

Explanation - From (I) six persons are in two rows of three persons each as shown below

From (IV) D is at the extreme left of one of the rows X and C is at the middle of row X From (II), F must be sitting in the other row and A and B are sitting in the same row as F. Hence, E must be in the same row as C. From (IV), A is sitting opposite E. D is sitting opposite B The final arrangement is as follows: E C D A F B A and D are at extreme ends of two different rows and also NOT opposite to each other.

I.25) Each of the critical reasoning questions are based on a short argument, a set of statements, or a plan of action. For each question, select the best answer of the choices given. Political theorist: Even with the best spies, area experts, and satellite surveillance, foreign policy assessments can still lack important information. In such circumstances intuitive judgment is vital. A national leader with such judgment can make good decisions about foreign policy even when current information is incomplete, since __________.

25. Which of the following, if true, most logically

completes the argument? a) the central reason for failure in foreign policy

decision making is the absence of critical information

b) those leaders whose foreign policy decisions have been highly ranked have also been found to have good intuitive judgment

c) both intuitive judgment and good information are required for sound decision making

d) good foreign policy decisions often lead to improved methods of gathering information

Page 28: Question Paper 2 RBI Grade B 2019 Phase 1 General ... · 4. Who is the brand ambassador of MasterCard? Ans. MS Dhoni [Covered in the Course? – Yes in October Spotlight] 5. Where

RBI Grade B 2019 Phase 1 – Reasoning 33

Prepare for RBI, SEBI & NABARD, UPSC & UGC NET Exam [email protected]; +91 9999466225

e) intuitive judgment can produce good decisions based on past experience, even when there are important gaps in current information Ans: e Explanation - Situation National leaders sometimes must make foreign policy decisions while lacking important information. Reasoning: What would most help support the claim that a national leader with intuitive judgment can make good foreign policy decisions without complete information? Th e word since preceding the blank indicates that the blank should be filled with a premise supporting the statement immediately before the blank. So an observation that supports the claim that a national leader with intuitive judgment can make good foreign policy decisions without complete information would logically complete the argument.

A. This gives us no reason to suppose that intuitive judgment helps national leaders avoid such failures.

B. This does not specify who ranked the foreign policy decisions, nor how they determined the rankings, so it gives us no reason to accept those rankings. For all we know, the anonymous rankers may have used the dubious rankings they created as the sole evidence for their so-called findings about which leaders have good intuitive judgment.

C. This implies that intuitive judgment alone is inadequate without good information, so it undermines rather than supports the claim that national leaders can make good foreign policy decisions with intuitive judgment while lacking complete information.

D. This gives us no reason to suppose that good foreign policy decisions can be made in the first place by leaders lacking important information.

E. Correct. This suggests that national leaders can make good foreign policy decisions using intuitive judgment based on their past foreign policy experience, even without complete information about the current situations they’re facing. The correct answer is E.

I.56) Each of the critical reasoning questions are based on a short argument, a set of statements, or a plan of action. For each question, select the best answer of the choices given. During the earliest period of industrialization in Britain, steam engines were more expensive to build and operate than either windmills or water mills, the other practicable sources of power for factories. Yet despite their significant cost disadvantage, steam‐powered factories were built in large numbers well before technical improvements brought their cost down. Furthermore, they were built even in regions where geographical conditions permitted the construction of wind‐ and water‐powered factories close to major markets.

26. Which of the following, if true, most helps to

explain the proliferation of steam‐powered factories during the earliest period of industrialization in Britain?

a) In many areas of Britain, there were fewer steam‐powered factories than wind‐ or water‐powered factories in the earliest period of industrialization.

b) Unlike wind‐ or water‐powered factories, steam‐powered factories were fuelled with coal, which sometimes had to be transported

Page 29: Question Paper 2 RBI Grade B 2019 Phase 1 General ... · 4. Who is the brand ambassador of MasterCard? Ans. MS Dhoni [Covered in the Course? – Yes in October Spotlight] 5. Where

RBI Grade B 2019 Phase 1 – Reasoning 34

Prepare for RBI, SEBI & NABARD, UPSC & UGC NET Exam [email protected]; +91 9999466225

significant distances from the mine to the site of the factory.

c) It was both difficult and expensive to convert a factory from wind power or water power to steam power.

d) In the early period of industrialization, many goods sold in towns and cities could not be mass‐produced in factories.

e) In Britain, the number of sites where a wind‐ or water‐powered factory could be built was insufficient to provide for all of the demand for factory‐produced goods at the time. Ans: e

Explanation - Argument Construction Situation Although steam engines were more expensive than windmills and water mills in early industrial Britain, many steam‐powered factories were built even in regions where the construction of wind‐and water‐powered factories was geographically feasible. Reasoning: Why might steam‐powered factories have proliferated despite their cost disadvantage? Early industrialists would have needed some positive reason to choose steam over less expensive power sources for their factories. For example, steam engines might have operated faster or more effectively than windmills or water mills. Or steam engines might have received government subsidies. Or conditions restricting the number or locations of windmills and water mills might have forced industrialists to use steam power instead.

A. This suggests that the steam‐powered factories did not initially proliferate as widely as they might have, but it does not explain why they proliferated to the extent that they did.

B. The inconvenience of transporting coal for steam‐powered factories would have made those factories less likely to proliferate, not more likely.

C. The difficulty of converting factories to steam power would have made steam‐powered factories less likely to proliferate, not more likely.

D. The technological inability to mass‐produce popular products in factories would have made factories in general less likely to proliferate, including steam‐powered factories.

E. Correct. The inadequate number of sites for wind‐ and water‐powered factories might have encouraged early industrialists to build steam‐powered factories instead, since the high demand for factory‐produced goods could have made these factories profitable despite their cost disadvantage. The correct answer is E.

I.57) Each of the critical reasoning questions are based on a short argument, a set of statements, or a plan of action. For each question, select the best answer of the choices given. Snowmaking machines work by spraying a mist that freezes immediately on contact with cold air. Because the sudden freezing kills bacteria, Quick Freeze is planning to market a wastewater purification system that works on the same principle. The process works only when temperatures are cold, however, so municipalities using it will still need to maintain a conventional system.

27. Which of the following, if true, provides the

strongest grounds for a prediction that municipalities will buy Quick Freeze’s purification system despite the need to

Page 30: Question Paper 2 RBI Grade B 2019 Phase 1 General ... · 4. Who is the brand ambassador of MasterCard? Ans. MS Dhoni [Covered in the Course? – Yes in October Spotlight] 5. Where

RBI Grade B 2019 Phase 1 – Reasoning 35

Prepare for RBI, SEBI & NABARD, UPSC & UGC NET Exam [email protected]; +91 9999466225

maintain a conventional purification system as well?

a) Bacteria are not the only impurities that must be removed from wastewater.

b) Many municipalities have old wastewater purification systems that need to be replaced.

c) Conventional wastewater purification systems have not been fully successful in killing bacteria at cold temperatures.

d) During times of warm weather, when it is not in use, Quick -Freeze’s purification system requires relatively little maintenance.

e) Places where the winters are cold rarely have a problem of water shortage. Ans: c

Explanation: Evaluation of a Plan Situation Quick-freeze is planning to market wastewater purification systems that work by spraying a mist that freezes on contact with cold air. Th e sudden freezing kills bacteria. Because the system works only at cold temperatures, municipalities using it will still need to maintain a conventional system. Reasoning: Which statement provides the strongest grounds for thinking that at least some municipalities will buy the purification system despite the need to maintain a conventional purification system as well? The passage tells us why a municipality using a Quick-Freeze wastewater purification system would still need a conventional system. But why would a municipality want the Quick-Freeze system in addition to a conventional system? If conventional systems are not fully effective at cold temperatures, the Quick-Freeze system would allow municipalities that sometimes experience cold temperatures to purify their wastewater more effectively.

A. There is no basis in the passage for determining whether the Quick-Freeze system will help remove impurities other than bacteria from wastewater. If it does not, this answer choice implies that the Quick-Freeze system would not be sufficient for purifying wastewater. Th is would actually undermine the prediction.

B. The passage states that municipalities using the Quick-Freeze system would still need a conventional system. Thus, the old conventional wastewater systems would still need to be replaced with new conventional systems. Th is answer choice provides no reason to think municipalities would buy the Quick-Freeze system.

C. Correct. Th is statement, if true, would strengthen the prediction, because it provides a valid reason why the Quick-Freeze system could be needed alongside conventional ones: it is more effective in cold weather.

D. Although this claim does undercut one reason for thinking municipalities might not be likely to purchase the Quick-Freeze system, it provides little reason to think that they will purchase such a system. Perhaps in times of cold weather, the Quick-Freeze system is very expensive to maintain.

E. The issue of whether or not there are water shortages in places where winters are cold is not directly relevant. If conventional wastewater systems are sufficient to purify water in such places, municipalities would not need the Quick-Freeze system (as they would still need to maintain a conventional purification system). The correct answer is C.

I.28) Each of the critical reasoning questions are based on a short argument, a set of statements, or a plan of action. For each

Page 31: Question Paper 2 RBI Grade B 2019 Phase 1 General ... · 4. Who is the brand ambassador of MasterCard? Ans. MS Dhoni [Covered in the Course? – Yes in October Spotlight] 5. Where

RBI Grade B 2019 Phase 1 – Reasoning 36

Prepare for RBI, SEBI & NABARD, UPSC & UGC NET Exam [email protected]; +91 9999466225

question, select the best answer of the choices given. Suncorp, a new corporation with limited funds, has been clearing large sections of the tropical Amazon forest for cattle ranching. This practice continues even though greater profits can be made from rubber tapping, which does not destroy the forest, than from cattle ranching, which does destroy the forest.

28. Which of the following, if true, most helps to

explain why Suncorp has been pursuing the less profitable of the two economic activities mentioned above?

a) The soil of the Amazon forest is very rich in nutrients that are important in the development of grazing lands.

b) Cattle‐ranching operations that are located in tropical climates are more profitable than cattle‐ranching operations that are located in cold‐weather climates.

c) In certain districts, profits made from cattle ranching are more heavily taxed than profits made from any other industry.

d) Some of the cattle that are raised on land cleared in the Amazon are killed by wildcats.

e) The amount of money required to begin a rubber‐tapping operation is twice as high as the amount needed to begin a cattle ranch. Ans: e

Explanation - Argument Construction Situation Suncorp is a new corporation with limited funds. It has been clearing large sections of the tropical Amazon forest for ranching, even though rubber‐tapping would be more profitable. Reasoning: What would explain why Suncorp is clearing sections of the rain forest for ranching,

even though rubber tapping would be more profitable? Because Suncorp has limited funds, if rubber tapping has much higher start‐up costs, Suncorp might not have enough money to start rubber‐tapping operations. If cattle ranching has much lower start‐up costs than rubber tapping, Suncorp might be able to afford such an operation.

A. This statement gives a reason why cattle

ranching in the Amazon might be more profitable than one might otherwise think it would be. However, we already know from the passage that rubber tapping would be more profitable than cattle ranching. So, this answer choice does not help explain why cattle ranching might be preferable to rubber tapping.

B. The comparison between the profitableness of cattle ranching in tropical climates and in cold‐weather climates is irrelevant. Th e passage only covers cattle ranching in the tropical Amazon forest. Th is answer choice would at most explain why Suncorp is undertaking cattle ranching in the Amazon rather than in some cold‐weather location.

C. This statement makes what needs to be explained harder to understand, for it indicates that cattle ranching in the Amazon might be less profitable than one would otherwise think.

D. Like answer choice (C), this statement indicates a disadvantage of cattle ranching in the Amazon. So, it does not explain why cattle ranching would be preferred to some other economic activity.

E. Correct. Because it costs less to begin cattle ranching than it does to begin rubber tapping, Suncorp— which has limited funds—would have a reason to pursue cattle ranching over a potentially more profitable activity. The correct answer is E.

Page 32: Question Paper 2 RBI Grade B 2019 Phase 1 General ... · 4. Who is the brand ambassador of MasterCard? Ans. MS Dhoni [Covered in the Course? – Yes in October Spotlight] 5. Where

RBI Grade B 2019 Phase 1 – Reasoning 37

Prepare for RBI, SEBI & NABARD, UPSC & UGC NET Exam [email protected]; +91 9999466225

I.29) Each of the critical reasoning questions are based on a short argument, a set of statements, or a plan of action. For each question, select the best answer of the choices given. Archaeologists use technology to analyse ancient sites. It is likely that this technology will advance considerably in the near future, allowing archaeologists to gather more information than is currently possible. If they study certain sites now, they risk contaminating or compromising them for future studies. Therefore, in order to maximize the potential for gathering knowledge in the long run, a team of archaeologists plans to delay the examination of a newly excavated site.

29. Which of the following would be most useful to

investigate for the purpose of evaluating the plan’s prospects for achieving its goal?

a) Whether any of the contents of the site will significantly deteriorate before the anticipated technology is available

b) Whether there will continue to be improvements on the relevant technology.

c) Whether the team can study a site other than the newly excavated site for the time being

d) Whether the site was inhabited by a very ancient culture

e) Whether the anticipated technology will damage objects under study Ans: a

Explanation - Evaluation of a Plan Situation: To avoid prematurely compromising a newly excavated site, an archaeological team plans to postpone examining it until more advanced technology is developed that will let

them gather more information from it. Their goal is to maximize the potential for gathering knowledge.

Reasoning: What would be most helpful to investigate in order to assess how likely it is that delaying examination of the site will maximize the potential for gathering knowledge from it? In order to maximize (or even increase) the potential for gathering knowledge from the site by delaying its examination, the risk of compromising the site by examining it now has to be greater than the risk that the site will be compromised as much or more by delaying the examination. Th e delay might also increase the risk that the site will never be examined at all—for example, the team might lose its funding while it delays, or changes in local political conditions might prevent the site’s future examination. Investigating any of these risks could be helpful in assessing the likelihood that the team’s plan will achieve its goal.

A. Correct. If any of the site’s contents will

significantly deteriorate before the technology becomes available, that could reduce the ability to gather future information from the site even more than examining and compromising the site now would.

B. The passage already tells us that it is likely the technology will advance considerably in the near future. Given this information, further inquiry into whether there will be any ongoing (perhaps minor) improvements is somewhat redundant and probably of minimal value with respect to evaluating the plan’s likelihood of success.

C. Even if the team can study a second site in the meanwhile, they might maximize the overall potential for gathering knowledge by delaying

Page 33: Question Paper 2 RBI Grade B 2019 Phase 1 General ... · 4. Who is the brand ambassador of MasterCard? Ans. MS Dhoni [Covered in the Course? – Yes in October Spotlight] 5. Where

RBI Grade B 2019 Phase 1 – Reasoning 38

Prepare for RBI, SEBI & NABARD, UPSC & UGC NET Exam [email protected]; +91 9999466225

the examination of either site, both sites, or neither site until more advanced technology is available.

D. The age of the culture that inhabited the site is irrelevant to assessing the risks of delaying the site’s examination until more advanced technology is available.

E. Even if the anticipated technology will damage or destroy the objects under study, it might still maximize the amount of knowledge that can be gathered from those objects. Without any comparison between the damage risk that would be incurred by proceeding with the current technology and the damage risk that would be incurred by waiting, the mere fact that some damage would occur is irrelevant. The correct answer is A.

I.30) Each of the critical reasoning questions are based on a short argument, a set of statements, or a plan of action. For each question, select the best answer of the choices given. Motorists in a certain country frequently complain that traffic congestion is much worse now than it was 20 years ago. No real measure of how much traffic congestion there was 20 years ago exists, but the motorists’ complaints are almost certainly unwarranted. The country’s highway capacity has tripled in the last twenty years, thanks to a vigorous highway construction program, whereas the number of automobiles registered in the country has increased by only 75 percent.

30. Which of the following, if true, most seriously

weakens the argument? a) Most automobile travel is local, and the

networks of roads and streets in the country’s

settled areas have changed little over the last 20 years.

b) Gasoline prices are high, and miles travelled per car per year have not changed much over the last 20 years.

c) The country’s urban centres have well‐developed public transit systems that carry most of the people who commute into those centres.

d) The average age of automobiles registered in the country is lower now than it was 20 years ago.

e) Radio stations have long been broadcasting regular traffic reports that inform motorists about traffic Ans: a

Explanation - Situation: Motorists complain that traffic congestion in their country is much worse than it was twenty years ago. But these complaints have no basis since the highway capacity in this country has tripled in the same period, whereas the number of cars registered has risen by only 75 percent. Reasoning: Which point most undermines the argument that the complaints are unwarranted? Consider that the response to the generalized complaints about congestion discusses only the topic of highway capacity. What if the congestion that motorists are complaining about is not on highways but on local roads? Discovering that travel tends to be local in this country and that the local roads have not been improved in the last twenty years would seriously weaken the argument.

A. Correct. This statement properly identifies a weakness in the argument: the response to the broad complaint addresses a different subject, highway capacity, not the issue of traffic congestion encountered by most motorists.

Page 34: Question Paper 2 RBI Grade B 2019 Phase 1 General ... · 4. Who is the brand ambassador of MasterCard? Ans. MS Dhoni [Covered in the Course? – Yes in October Spotlight] 5. Where

RBI Grade B 2019 Phase 1 – Reasoning 39

Prepare for RBI, SEBI & NABARD, UPSC & UGC NET Exam [email protected]; +91 9999466225

B. If high gas prices actually prevented motorists from driving, and if motorists’ driving habits were the same as they were twenty years ago, then these points should strengthen the argument that there is no basis for their complaints.

C. The number of commuters who use public transit does not affect the argument that the motorists’ complaints have no basis.

D. The age of registered cars is irrelevant to the argument.

E. The radio broadcasts attest to the existence of traffic, but not to its increase, so they do not affect the argument. The correct answer is A.

I.31) Each of the critical reasoning questions are based on a short argument, a set of statements, or a plan of action. For each question, select the best answer of the choices given. The percentage of households with an annual income of more than $40,000 is higher in Merton County than in any other county. However, the percentage of households with an annual income of $60,000 or more is higher in Sommer County.

31. If the statements above are true, which of the

following must also be true?

a) The percentage of households with an annual income of $80,000 is higher in Sommer County than in Merton County.

b) Merton County has the second highest percentage of households with an annual income of $60,000 or more.

c) Some households in Merton County have an annual income between $40,000 and $60,000.

d) The number of households with an annual income of more than $40,000 is greater in Merton County than in Sommer County.

e) Average annual household income is higher in Sommer County than in Merton County. Ans: c

Explanation - Situation: The percentage of households with annual incomes of more than $40,000 is higher in Merton County than in any other county; the percentage of households with annual incomes of $60,000or more is higher in Sommer County. Reasoning On the basis of this information, what point must be true? The given information makes clear that Merton County has some households that exceed $40,000 in annual income. Sommer County has a higher percentage of households with annual incomes at or above $60,000. A higher percentage of the Merton County households must in turn have annual incomes of $60,000 or less. Thus, the annual income of some households in Merton County is between $40,000 and $60,000.

A. Since it is possible that there are no households with an annual income of $80,000 in Sommer County, this statement does not follow from the situation.

B. It is not possible to make this determination on the basis of the available evidence; Merton County may, have no households at all with an income of more than $60,000.

C. Correct. This statement properly identifies a conclusion that can be drawn from the given information: in order for the percentage of $40,000‐plus incomes to be higher in Merton county than any other county while Sommer has the highest percentage of $60,000‐plus incomes, there must be some households in

Page 35: Question Paper 2 RBI Grade B 2019 Phase 1 General ... · 4. Who is the brand ambassador of MasterCard? Ans. MS Dhoni [Covered in the Course? – Yes in October Spotlight] 5. Where

RBI Grade B 2019 Phase 1 – Reasoning 40

Prepare for RBI, SEBI & NABARD, UPSC & UGC NET Exam [email protected]; +91 9999466225

Merton County that bring in between $40,000 and $60,000 annually.

D. On the basis of information about the percentages of households, it is not possible to arrive at this conclusion about the number of households.

E. From the given information, it is not possible to determine where the average income is greater. It is entirely possible that the number of $60,000‐plus incomes in Sommer County is quite small and that the number of $40,000‐plus incomes in Merton County is substantial. The correct answer is C.

32. If ‘BOMBAY’ is written as ‘MYMYMY’, then how

will ‘TAMILNADU’ be written in that code?

a) TIATIATIA b) MNUMNUMNU c) IATIATIAT d) ALDALDA

Ans: b Explanations – The letters at the third and sixth place are repeated thrice to code BOMBAY as MYMYMY. Similarly, the letters at the third, sixth and ninth places are repeated thrice to code TAMILNADU as MNUMNUMNU.

33. If ‘PEN’ and ‘MONEY’ are coded as ‘GEE’ and

‘DFEBEG’ then ‘MAGGY’ will be coded as:

a) DAYYY b) DYYYG c) DAYYG d) DAGGG

Ans: d

Explanations - Let A, B, C….Z be represented by the number 1, 2, 3......26 respectively. Then

P (16) E (5) N(14) = G (7) E(5) E (5) M (13) O (15) N (14) K (11) E (5) Y (25) = D (4) F (6) E (5) B(2) E(5) G (7) Hence, M (13) A (1) G (7) G (7) Y (25) = D (4) A (1) G(7) G (7) G (7) As single digit numbers do not change while for double-digit numbers, the digits are added. Hence (d).

34. ‘I am smart’ is coded as ‘spjm okz pkj’ and ‘I

know everything’ is coded as ‘sat zyk okz’.Which of the following could be the code of ‘ smart girls know everything’?

a) pkj spjm gre ykz b) sat zyk spjm pkj c) sat gre cat pkj d) pkj cat zyk sat e) Ans: d

Explanation – Code for ‘I’ and ‘SMART’ are ‘SPJM’ and ‘PKJ’. Code for ‘KNOW’ and ‘EVERYTHING’ are ‘SAT’ and ‘ZYK’. Hence in the code for the given sentence ‘SMART GIRLS KNOW EVERYTHING’ both the codes ‘SAT’ and ‘ZYK’ and only one from ‘SPJM’ and ‘PKJ’ should be used.

35. Each question consists of a pair of terms related in a certain way, followed by another pair of terms. Choose the alternative such that the known term bears the same relation to the unknown term represented by (?) in the second pair as the relation between the two terms in the first pair.

Page 36: Question Paper 2 RBI Grade B 2019 Phase 1 General ... · 4. Who is the brand ambassador of MasterCard? Ans. MS Dhoni [Covered in the Course? – Yes in October Spotlight] 5. Where

RBI Grade B 2019 Phase 1 – Reasoning 41

Prepare for RBI, SEBI & NABARD, UPSC & UGC NET Exam [email protected]; +91 9999466225

DALE: LEAD :: PALE: ?

a) PEAL b) LEAP c) APEL d) ELAP

Ans: b

Explanation – The 1st, 2nd, 3rd and 4th letters of the second group are respectively the 4th, 3rd, 1st and 2nd letters of the first group.

36. Each question consists of a pair of terms

related in a certain way, followed by another pair of terms. Choose the alternative such that the known term bears the same relation to the unknown term represented by (?) in the second pair as the relation between the two terms in the first pair.

WAYLAY: AEIOUI :: LAYWAY : ?

a) OUIAEI b) UAIOEI c) EOUIIA d) AEIOUI

Ans: a

Explanation – There is no particular relationship between the letters. The first group of letters are matched with the next group. Hence, W = A, A=E, Y=I, L=O, A= U and Y=I.

37. Each question consists of a pair of terms

related in a certain way, followed by another pair of terms. Choose the alternative such that the known term bears the same relation to the unknown term represented by (?) in the second pair as the relation between the two terms in the first pair.

BACE: DACE: FACE:?

a) HASE b) LACE c) CASE d) HACE

Ans: d

Explanation – Each term has ACE common in all. Between consonants, one is skipped that is after B, C is skipped and next term starts with D. After D, E is skipped and E starts, and after G is skipped and H starts.

38. Each question consists of a pair of terms

related in a certain way, followed by another pair of terms. Choose the alternative such that the known term bears the same relation to the unknown term represented by (?) in the second pair as the relation between the two terms in the first pair.

CQGP: YNEO :: UTPR: ?

a) QNQQ b) TUWV c) TPQR d) QQNQ

Ans: d

Explanation – The 1st, 2nd, 3rd and 4th letters of the first group are moved 4, 3, 2 and 1 step backward to get the letters of the second group

I.39) Three dacoits D1, D2 and D3 each has been terrorizing three different men, M1, M2and M3 not necessarily in that order. Each of these men is a citizen of three different areas A1, A2 and A3 not necessarily in that order. Each of the dacoit favors a different mode of

Page 37: Question Paper 2 RBI Grade B 2019 Phase 1 General ... · 4. Who is the brand ambassador of MasterCard? Ans. MS Dhoni [Covered in the Course? – Yes in October Spotlight] 5. Where

RBI Grade B 2019 Phase 1 – Reasoning 42

Prepare for RBI, SEBI & NABARD, UPSC & UGC NET Exam [email protected]; +91 9999466225

transport, horses, jeeps and boats, again not necessarily in that order. D2 and his horsemen do not terrorize M3, D3 is not water borne and does not operate in A2. Neither M3 nor M2 has anything to do with A3 which has been terrorized by dacoits in jeeps. D1 is not from area A1.

39. D3’s mode of transport is: a) Horse b) Jeep c) Boat d) Cannot be determined

Ans: b

Explanation – Step 1 Identif

y the Dacoits

D1 D2 D3

Step 2 Identify the Men

M1 M2 M3

Step 3 Identify the Areas

A1 A2 A3

Step 4 Identify the Modes of transport

Horses Jeeps Boats

Conditions

A. D2’s mode of transport is horse and does not terrorize M3. D2-> Horses, D2-> M3

B. D3 does not operate in A2 and his mode of transport is not the boat. D3 -> Boat x, D3 -> A2 M3 and M2 do not belong to A3 and dacoits of this region travel by jeeps.

M3 -> A3, M2 -> A3, A3-> Jeep Using the conditions, we get

Type of Transpo

rt

Men Area

D1 Boat M3 A2 D2 horse M2 A1 D3 jeep M1 A3

I.40) Read the data below and answer the questions that follow. Three dacoits D1, D2 and D3 each has been terrorizing three different men, M1, M2and M3 not necessarily in that order. Each of these men is a citizen of three different areas A1, A2 and A3 not necessarily in that order. Each of the dacoit favors a different mode of transport, horses, jeeps and boats, again not necessarily in that order. D2 and his horsemen do not terrorize M3, D3 is not water borne and does not operate in A2. Neither M3 nor M2 has anything to do with A3 which has been terrorized by dacoits in jeeps. D1 is not from area A1.

40. M3 has been terrorized by: a) D1 b) D2 c) D3 d) Cannot be determined.

Ans: a

Explanation - Step 1

Identify the Dacoits

D1 D2 D3

Step 2

Identify the Men

M1 M2 M3

Page 38: Question Paper 2 RBI Grade B 2019 Phase 1 General ... · 4. Who is the brand ambassador of MasterCard? Ans. MS Dhoni [Covered in the Course? – Yes in October Spotlight] 5. Where

RBI Grade B 2019 Phase 1 – Reasoning 43

Prepare for RBI, SEBI & NABARD, UPSC & UGC NET Exam [email protected]; +91 9999466225

Step 3

Identify the Areas

A1 A2 A3

Step 4

Identify the modes of transport

Horses Jeeps Boats

Conditions

A. D2’s mode of transport is horse and does not terrorize M3. D2-> Horses, D2-> M3

B. D3 does not operate in A2 and his mode of transport is not the boat. D3 -> Boat x, D3 -> A2 M3 and M2 do not belong to A3 and dacoits of this region travel by jeeps. M3 -> A3, M2 -> A3, A3-> Jeep Using the conditions, we get Type of

Transport

Men Area

D1 Boat M3 A2

D2 horse M2 A1

D3 jeep M1 A3

I.41) Read the data below and answer the questions that follow. Three dacoits D1, D2 and D3 each has been terrorizing three different men, M1, M2and M3 not necessarily in that order. Each of these men is a citizen of three different areas A1, A2 and A3 not necessarily in that order. Each of the dacoit favors a different mode of transport, horses, jeeps and boats, again not necessarily in that order. D2 and his horsemen do not terrorize M3, D3 is not water borne and does

not operate in A2. Neither M3 nor M2 has anything to do with A3 which has been terrorized by dacoits in jeeps. D1 is not from area A1.

41. The bandit on horseback is the terror of: a) A2 b) A3 c) A1 d) Cannot be determined

Ans: c Explanation -

Step 1 Identify the Dacoits

D1 D2 D3

Step 2 Identify the Men

M1 M2 M3

Step 3

Identify the Areas

A1 A2 A3

Step 4

Identify the modes of transport

Horses Jeeps Boats

Conditions A. D2’s mode of transport is horse and does not

terrorize M3. D2-> Horses, D2-> M3

B. D3 does not operate in A2 and his mode of transport is not the boat. D3 -> Boat x, D3 -> A2 M3 and M2 do not belong to A3 and dacoits of this region travel by jeeps. M3 and M2 do not belong to A3 and dacoits of this region travel be jeeps. M3 -> A3, M2 -> A3, A3-> Jeep Using the conditions, we get

Page 39: Question Paper 2 RBI Grade B 2019 Phase 1 General ... · 4. Who is the brand ambassador of MasterCard? Ans. MS Dhoni [Covered in the Course? – Yes in October Spotlight] 5. Where

RBI Grade B 2019 Phase 1 – Reasoning 44

Prepare for RBI, SEBI & NABARD, UPSC & UGC NET Exam [email protected]; +91 9999466225

Type of Transport

Men Area

D1 Boat M3 A2

D2 horse M2 A1

D3 jeep M1 A3

I.42) Read the data below and answer the questions that follow. Three dacoits D1, D2 and D3 each has been terrorizing three different men, M1, M2and M3 not necessarily in that order. Each of these men is a citizen of three different areas A1, A2 and A3 not necessarily in that order. Each of the dacoit favors a different mode of transport, horses, jeeps and boats, again not necessarily in that order. D2 and his horsemen do not terrorize M3, D3 is not water borne and does not operate in A2. Neither M3 nor M2 has anything to do with A3 which has been terrorized by dacoits in jeeps. D1 is not from area A1.

42. D3 has been terrorizing: a) M3 b) M1 c) M2 d) Cannot be determined

Ans: b

Step 1 Identify the

Dacoits

D1 D2 D3

Step 2 Identify the Men

M1 M2 M3

Step 3 Identify the Areas

A1 A2 A3

Step 4 Identify the

modes of

transport

Horses Jeeps Boats

Conditions

A. D2’s mode of transport is horse and does not terrorize M3. D2-> Horses, D2-> M3

B. D3 does not operate in A2 and his mode of transport is not the boat. D3 -> Boat x, D3 -> A2 M3 and M2 do not belong to A3 and dacoits of this region travel by jeeps. M3 and M2 do not belong to A3 and dacoits of this region travel be jeeps. M3 -> A3, M2 -> A3, A3-> Jeep Using the conditions, we get

Type of Transport

Men Area

D1 Boat M3 A2

D2 horse M2 A1

D3 jeep M1 A3

I.43) Read the data below and answer the questions that follow. Three dacoits D1, D2 and D3 each has been terrorizing three different men, M1, M2and M3 not necessarily in that order. Each of these men is a citizen of three different areas A1, A2 and A3 not necessarily in that order. Each of the dacoit favors a different mode of transport, horses, jeeps and boats, again not necessarily in that order. D2 and his horsemen do not terrorize M3, D3 is not water borne and does not operate in A2. Neither M3 nor M2 has

Page 40: Question Paper 2 RBI Grade B 2019 Phase 1 General ... · 4. Who is the brand ambassador of MasterCard? Ans. MS Dhoni [Covered in the Course? – Yes in October Spotlight] 5. Where

RBI Grade B 2019 Phase 1 – Reasoning 45

Prepare for RBI, SEBI & NABARD, UPSC & UGC NET Exam [email protected]; +91 9999466225

anything to do with A3 which has been terrorized by dacoits in jeeps. D1 is not from area A1.

43. A2 has been terrorized by: a) D1 b) D2 c) D3 d) Cannot be determined

Ans: a

Explanation: Step

1 Identify

the Dacoits

D1 D2 D3

Step 2

Identify the

Men

M1 M2 M3

Step 3

Identify the

Areas

A1 A2 A3

Step 4

Identify the

modes of

transport

Horses Jeeps Boats

Conditions

A. D2’s mode of transport is horse and does not terrorize M3. D2-> Horses, D2-> M3

B. D3 does not operate in A2 and his mode of transport is not the boat. D3 -> Boat x, D3 -> A2 M3 and M2 do not belong to A3 and dacoits of this region travel by jeeps. M3 and M2 do not belong to A3 and dacoits of this region travel be jeeps. M3 -> A3, M2 -> A3, A3-> Jeep Using the conditions, we get

Type of Transport

Men Area

D1 Boat M3 A2

D2 horse M2 A1

D3 jeep M1 A3

I.44-48) Read the data below and answer the questions that follow. Geological survey of India will conduct a survey in the Himalayan range. Two teams of minimum three people will be sent for doing the survey. There are seven men, A, B, C, D, E, F and G who will go for the survey. The men are experts in one of the fields out of map surveying, Data Interpretation, Paleontology and Seismology. Every group must have at least one Paleontologist and one Seismologist. The formation of groups follows few conditions.

• A and B are map surveyors and they cannot be together

• D and E are Seismologists • C and G cannot be together, and G and B are

always in the same group. • F is a paleontologist.

44. If F is in the same group with B, then C must be

specialized in which field? a) Data Interpretation b) Paleontology c) Seismology d) Map Surveying

Ans: b Explanation -

Group I Group II

A (MS) B(MS)

Page 41: Question Paper 2 RBI Grade B 2019 Phase 1 General ... · 4. Who is the brand ambassador of MasterCard? Ans. MS Dhoni [Covered in the Course? – Yes in October Spotlight] 5. Where

RBI Grade B 2019 Phase 1 – Reasoning 46

Prepare for RBI, SEBI & NABARD, UPSC & UGC NET Exam [email protected]; +91 9999466225

E/D (S) D/E (S)

C G

F can be in any of the groups If F is in group II, then C must be a Paleontologist as there should be at least one Paleontologist as there should be at least one Paleontologist in each group.

45. If F cannot go, then what must be the

specialization of G? a) Paleontology b) Seismology c) Data Interpretation d) Map Surveying

Ans: a

Explanation - Group I

Group II A (MS) B(MS)

E/D (S) D/E (S)

C G

If F cannot go then both C and G must be Paleontologists.

46. If D cannot be in the same group of F, then who

must always be in the group of F? a) A b) B c) C d) E

Ans: d Explanation -

Group I Group II A (MS) B(MS) E/D (S) D/E (S)

C G If D and F are not in the same group, then the group of F must have E, as both the groups must always have one Seismologist.

47. If C is in the field of Data Interpretation, then

who must always be in the same group of C? a) B b) D c) F d) E

Ans: c

Explanation - Group I Group II

A (MS) B(MS)

E/D (S) D/E (S)

C G

If C is an Data Interpreter, then F must be in his group as at least one Paleontologist must be there in each group.

48. If B is not in any of the group, then who can be

in the same group of A? a) G and E b) C and E c) C and F d) (a) or (b) or (c)

Ans: d

Explanation – If B is not there, then there is no restriction for G: Hence, G can be with A. Also, any of E and D can be with A. The same for F.

I.49-51) In a two -generation family, there are nine members. There are four males Rakesh, Ramesh, Suresh and Rohan out of which only two are married.

• Rakesh is Ramesh’s brother in law

Page 42: Question Paper 2 RBI Grade B 2019 Phase 1 General ... · 4. Who is the brand ambassador of MasterCard? Ans. MS Dhoni [Covered in the Course? – Yes in October Spotlight] 5. Where

RBI Grade B 2019 Phase 1 – Reasoning 47

Prepare for RBI, SEBI & NABARD, UPSC & UGC NET Exam [email protected]; +91 9999466225

• Suresh is Usha’s brother in law and Usha is not Rakesh’s wife

• Asha and Usha are not sisters • Rohan is the only nephew of Suresh and Asha • Manasi is Sheela’s cousin sister and Sheela is Ramesh’s

daughter. • Shobha is Asha’s daughter.

49. Who is Rohan’s sister? a) Sheela b) Manasi c) Shobha d) Cannot be determined

Ans: a

Explanation – All the members of the family are mentioned in the given data. From the data, we can say that Rohan, Manasi, Shobha and Sheela are from the second generation. As Usha is not Rakesh’s wife, she has to be Ramesh’s wife and thus Suresh will be an unmarried man and her brother in law, in the first generation. Usha is married to Ramesh and his brother Suresh is unmarried and his sister Asha is married to Rakesh. Ramesh is married to Usha and their children are Rohan and Sheela. While Rakesh and Asha have Shobha and Mansi as their daughters.

50. Which of the following is the correct pair of

siblings? a) Rakesh – Usha b) Ramesh – Usha c) Shobha- Manasi d) Rohan – Manasi

Ans: c

Explanation - All the members of the family are mentioned in the given data. From the data, we can say that Rohan, Manasi, Shobha and Sheela are from the second generation. As Usha is not Rakesh’s wife, she has to be Ramesh’s wife and thus Suresh will be an unmarried man and her brother in law, in the first generation. Usha is married to Ramesh and his brother Suresh is unmarried and his sister Asha is married to Rakesh. Ramesh is married to Usha and their children are Rohan and Sheela. While Rakesh and Asha have Shobha and Mansi as their daughters.

51. Shobha is the daughter of: a) Suresh b) Ramesh c) Rakesh d) Cannot be determined

Ans: c

Explanation - All the members of the family are mentioned in the given data. From the data, we can say that Rohan, Manasi, Shobha and Sheela are from the second generation. As Usha is not Rakesh’s wife, she has to be Ramesh’s wife and thus Suresh will be an unmarried man and her brother in law, in the first generation. Usha is married to Ramesh and his brother Suresh is unmarried and his sister Asha is married to Rakesh. Ramesh is married to Usha and their children are Rohan and Sheela. While Rakesh and Asha have Shobha and Mansi as their daughters.

52. Pick the odd one out in the group

Page 43: Question Paper 2 RBI Grade B 2019 Phase 1 General ... · 4. Who is the brand ambassador of MasterCard? Ans. MS Dhoni [Covered in the Course? – Yes in October Spotlight] 5. Where

RBI Grade B 2019 Phase 1 – Reasoning 48

Prepare for RBI, SEBI & NABARD, UPSC & UGC NET Exam [email protected]; +91 9999466225

a) 244 b) 686 c) 301 d) 581

Ans: a Explanation – The numbers in all option sets are divisible by 7 except 244. Hence (a)

53. Pick the odd one out in the group

2,2,3,4,6,9,15,22,35 a) 9 b) 6 c) 22 d) 15

Ans: d

Explanation – The Fibonacci series is as follows 1,1,2,3,5,8,13,21,34, ..... Adding 1 to each of the terms of the above series we get 2,2,3,4,6,9,14,22,35 Hence, 15 is the odd one out.

I.54-58) A numerical machine accepts two values X and Y. Then it updates these values as X= XY (product of X and Y) and Y = Y+1 in every step. The machine stops at X ≥ N.

54. For X=3, Y=2 and N=100, how many steps are

performed before the machine stops? a) 2 b) 3 c) 4 d) 5

Ans: c

Explanation – For these kinds of questions, one should make a table of values of X and Y in different steps.

X Y Input 3 2 Step1 6 3

Step 2 18 4 Step 3 72 5 Step 4 360 6

Machine stops at this step as 360> 100

55. In the above question, what is the final value of X?

a) 6 b) 20 c) 72 d) 360

Ans: d Explanation – For these kinds of questions, one should make a table of values of X and Y in different steps.

X Y

Input 3 2

Step1 6 3

Step 2 18 4

Step 3 72 5

Step 4 360 6

Final value of X =360

56. In the above question, what is the final value of

Y? a) 4 b) 5 c) 6 d) 20

Ans: c

Explanation – For these kinds of questions, one should make a table of values of X and Y in different steps.

X Y

Input 3 2

Page 44: Question Paper 2 RBI Grade B 2019 Phase 1 General ... · 4. Who is the brand ambassador of MasterCard? Ans. MS Dhoni [Covered in the Course? – Yes in October Spotlight] 5. Where

RBI Grade B 2019 Phase 1 – Reasoning 49

Prepare for RBI, SEBI & NABARD, UPSC & UGC NET Exam [email protected]; +91 9999466225

Step1 6 3

Step 2 18 4

Step 3 72 5

Step 4 360 6

Final Value of Y =6

57. If the value of N is changed to 500, then what

would be the final value of X in above question?

a) 360 b) 500 c) 560 d) 2160

Ans: d

Explanation – n =500 X Y

Input 3 2

Step 1 6 3

Step 2 18 4

Step 3 72 5

Step 4 360 6

Step 5 2160 7

Final value of X = 2160

58. If X=2 and Y=3, then what should be the

minimum value of N such that final value of Y is 7?

a) 300 b) 360 c) 720 d) 860

Ans: c

X Y

Input 2 3

Step 1 6 4

Step 2 24 5

Step3 120 6

Step 4 720 7

Hence, minimum value of N should be 720.

59. In a certain language, HUNIKA is coded as

93223 and TARITA is coded as 212127. What will be the code of NIVEDITA in this language?

a) 1529318 b) 2915429 c) 1429319 d) 1415429 e) 1529319

Ans: e

Explanation – As per the position of alphabets in the English language, H=8,U=21,N=14, I=9, K=11, A=1 Now for the next step, just add the first and the last number, then the second and the second last number and so on till we get the middle number. So for HUNIKA we have: HUNIKA = 8+1, 21+11, 14+9 = 93223 Similarly, T=20, A=1, R=18, I =9, T=20, A=1 Therefore, the codes for TARITA becomes TARITA= 20+1, 1+20, 18+9 =212127 Finally, the code for NIVEDITA becomes, N=14, I= 9, V=22, E=5, D=4, I=9, T=20,A=1 NIVEDITA = 14+1, 9+20, 22+9, 5+4 = 1529319

60. If X is the father of Y and Z, while Z is also B’s

brother and A is Y’s mother, then, which of the following cannot necessarily be true?

a) A is the mother of B b) B is the brother of Z

Page 45: Question Paper 2 RBI Grade B 2019 Phase 1 General ... · 4. Who is the brand ambassador of MasterCard? Ans. MS Dhoni [Covered in the Course? – Yes in October Spotlight] 5. Where

RBI Grade B 2019 Phase 1 – Reasoning 50

Prepare for RBI, SEBI & NABARD, UPSC & UGC NET Exam [email protected]; +91 9999466225

c) X is the father of B d) Z is the brother of Y e) Z is the son of A

Ans: b

Page 46: Question Paper 2 RBI Grade B 2019 Phase 1 General ... · 4. Who is the brand ambassador of MasterCard? Ans. MS Dhoni [Covered in the Course? – Yes in October Spotlight] 5. Where

RBI Grade B 2019 Phase 1 – Reasoning 51

Prepare for RBI, SEBI & NABARD, UPSC & UGC NET Exam [email protected]; +91 9999466225

I.1-4) The multi-layered pie-chart below shows the sales of LED television sets for a big retail electronics outlet during 2016 and 2017. The outer layer shows the monthly sales during this period, with each label showing the month followed by the sales figure of that month. For some months, the sales figures are not given in the chart. The middle-layer shows quarter-wise aggregate sales figures (in some cases, aggregate quarter-wise sales numbers are not given next to the quarter). The innermost layer shows annual sales. It is known that the sales figures during the three months of the second quarter (April, May, June) of 2016 form an arithmetic progression, as do the three monthly sales figures in the fourth quarter (October, November, December) of that year.

1. What is the percentage increase in sales in December 2017 as compared to the sales in December 2016?

a) 28.57 b) 22.22 c) 50.00 d) 38.46

Ans: a Explanation: For 2016

Month Monthly Sales Quarterly Sales Yearly sales

January 80 Jan + Feb +

March = 240 January +

February + March +

April + May + June +

July + August +

September + October + November

+ December

= 240+ 150+ 250 + 360 = 1000

February 60 March 100 April 40

April + May + June = 150

May X = 50

June 110-X = 60

July 75 July + August + September = 2

50

August 120 Septemb

er 55

October 100 October +

November + December =

360

November Y= 120

December

260 – Y = 140

Now we know that Sales in April, May and June together is 150. Now, we are given that x and 110 –x are in AP. So, their possible values are: 40 + x + 110 –X = 150 X- 40 = 110 – x – x 3X= 150 X =50 Now we know that Sales in October, November and December together was 360. Now, we are given that Y, 260 – Y and 100 are in AP. Y-100 = 260 –Y –Y 3Y = 360 Y = 120 For 2017

Month Monthly Sales Quarterly Sales Yearly sales

January 120 Jan + Feb +

March = 380

January + February + March +

April + May + June +

July +

February 100 March 160 April 60

April + May + June = 200 May 75

June 65

Page 47: Question Paper 2 RBI Grade B 2019 Phase 1 General ... · 4. Who is the brand ambassador of MasterCard? Ans. MS Dhoni [Covered in the Course? – Yes in October Spotlight] 5. Where

RBI Grade B 2019 Phase 1 – Reasoning 52

Prepare for RBI, SEBI & NABARD, UPSC & UGC NET Exam [email protected]; +91 9999466225

July 60

July + August + September = 2

20

August + September + October + November

+ December = 380 + 200 + 220 + 500

= 1300

August (220

– 130) = 90

September 70

October 150 October +

November + December = 50

0

November 170

December

(500 – 320 = 180)

Sales in December, 2016 = 140 Sales in December, 2017 = 180 Percentage increase in sales in 2017 as compared to sales in 2016. {(180 –140) / 140} * 100 = 40/ 140 = 28.57%

2. In which quarter of 2017 was the percentage increase in sales from the same quarter of 2016 the highest?

a) Q1 b) Q3 c) Q4 d) Q2

Ans: a Explanation: For 2016

Month Monthly Sales Quarterly Sales Yearly sales

January 80 Jan + Feb +

March = 240

January + February +

March + April + May

+ June + July +

August + September + October +

February 60 March 100 April 40

April + May + June = 150

May X = 50

June 110-X = 60

July 75 August 120

September 55

July + August + September = 2

50

November +

December = 240+

150+ 250 + 360 = 1000

October 100 October +

November + December =

360

November Y= 120

December

260 – Y = 140

Now we know that Sales in April, May and June together is 150. Now, we are given that x and 110 –x are in AP. So, their possible values are: 40 + x + 110 –X = 150 X- 40 = 110 – x – x 3X= 150 X =50 Now we know that Sales in October, November and December together was 360. Now, we are given that Y, 260 – Y and 100 are in AP. Y-100 = 260 –Y –Y 3Y = 360 Y = 120 For 2017

Month Monthly Sales Quarterly Sales Yearly sales

January 120 Jan + Feb +

March = 380

January + February + March +

April + May + June +

July + August +

September + October + November

+ December = 380 + 200 + 220 + 500

= 1300

February 100 March 160 April 60

April + May + June = 200 May 75

June 65 July 60

July + August + September = 2

20

August (220

– 130) = 90

September 70

October 150 October +

November + November 170

Page 48: Question Paper 2 RBI Grade B 2019 Phase 1 General ... · 4. Who is the brand ambassador of MasterCard? Ans. MS Dhoni [Covered in the Course? – Yes in October Spotlight] 5. Where

RBI Grade B 2019 Phase 1 – Reasoning 53

Prepare for RBI, SEBI & NABARD, UPSC & UGC NET Exam [email protected]; +91 9999466225

December

(500 – 320 = 180)

December = 500

Percentage comparison of quarters in two years

2016 2017 Percentage

increase from same quarter

Quarter 1 = 240

Quarter 1 = 380

Quarter 1 = 140 / 240 = 58.33

Quarter 2 = 150

Quarter 2= 200

Quarter 2 = 50/150 = 33.33

Quarter 3 = 250

Quarter 3 = 220

Quarter 3 = 30/250 = -12

Quarter 4 = 360

Quarter 4 = 500

Quarter 4 = 140/360 = 38.88

3. During which quarter was the percentage

decrease in sales from the previous quarter’s sales the highest?

a) Q2 of 2017 b) Q1 of 2017 c) Q4 of 2017 d) Q2 of 2016

Ans: a Explanations- For 2016

Month Monthly Sales Quarterly Sales Yearly sales

January 80 Jan + Feb +

March = 240 January +

February + March +

April + May + June +

July + August +

September + October + November

+ December

= 240+

February 60 March 100 April 40

April + May + June = 150

May X = 50

June 110-X = 60

July 75 July + August + September = 2

50

August 120 Septemb

er 55

October 100 October +

November + November Y= 120

December

260 – Y = 140

December = 360

150+ 250 + 360 = 1000

Now we know that Sales in April, May and June together is 150. Now, we are given that x and 110 –x are in AP. So, their possible values are: 40 + x + 110 –X = 150 X- 40 = 110 – x – x 3X= 150 X =50 Now we know that Sales in October, November and December together was 360. Now, we are given that Y, 260 – Y and 100 are in AP. Y-100 = 260 –Y –Y 3Y = 360 Y = 120 For 2017

Month Monthly Sales Quarterly Sales Yearly sales

January 120 Jan + Feb +

March = 380 January + February + March +

April + May + June +

July + August +

September + October + November

+ December = 380 + 200 + 220 + 500

= 1300

February 100 March 160 April 60

April + May + June = 200 May 75

June 65 July 60

July + August + September = 2

20

August (220

– 130) = 90

September 70

October 150 October +

November + December = 50

0

November 170

December

(500 – 320 = 180)

Using options given to us we will see that Option A) Q2 of 2017 Q1 of 2017 = 380

Page 49: Question Paper 2 RBI Grade B 2019 Phase 1 General ... · 4. Who is the brand ambassador of MasterCard? Ans. MS Dhoni [Covered in the Course? – Yes in October Spotlight] 5. Where

RBI Grade B 2019 Phase 1 – Reasoning 54

Prepare for RBI, SEBI & NABARD, UPSC & UGC NET Exam [email protected]; +91 9999466225

Q2 of 2017 = 200 Decrease from previous quarter = (180) / 380 = (9/16)* 100 = 225/4 = 56.25% Option B) Q1 of 2017 = 380 Q4 of 2016 = 360 There is no decrease but increase from previous quarter, so it can be ruled out. Option C) Q4 of 2017 = 500 Q3 of 2017 = 220 There is no decrease but increase from previous quarter. Hence, it can be ruled out. Option D) Q2 of 2016 = 150 Q1 of 2016 = 240 Decrease from previous quarter = 110 / 240 = 275/ 11 = 25% So, sales of Q2 of 2017 has highest decrease.

4. During which month was the percentage increase in sales from the previous month’s sales the highest?

a) March of 2017 b) October of 2017 c) October of 2016 d) March of 2016

Ans: b Explanation - For 2016

Month Monthly Sales Quarterly Sales Yearly sales

January 80 Jan + Feb +

March = 240

January + February +

March + April + May

+ June +

February 60 March 100 April 40 April + May +

June = 150 May X = 50

June 110-X = 60

July + August +

September + October + November

+ December

= 240+ 150+ 250 + 360 = 1000

July 75 July + August + September = 2

50

August 120 Septemb

er 55

October 100 October +

November + December =

360

November Y= 120

December

260 – Y = 140

Now we know that Sales in April, May and June together is 150. Now, we are given that x and 110 –x are in AP. So, their possible values are: 40 + x + 110 –X = 150 X- 40 = 110 – x – x 3X= 150 X =50 Now we know that Sales in October, November and December together was 360. Now, we are given that Y, 260 – Y and 100 are in AP. Y-100 = 260 –Y –Y 3Y = 360 Y = 120 For 2017

Month Monthly Sales Quarterly Sales Yearly sales

January 120 Jan + Feb +

March = 380

January + February + March +

April + May + June +

July + August +

September + October + November

+ December = 380 + 200

February 100 March 160 April 60

April + May + June = 200 May 75

June 65 July 60

July + August + September = 2

20

August (220

– 130) = 90

September 70

October 150

Page 50: Question Paper 2 RBI Grade B 2019 Phase 1 General ... · 4. Who is the brand ambassador of MasterCard? Ans. MS Dhoni [Covered in the Course? – Yes in October Spotlight] 5. Where

RBI Grade B 2019 Phase 1 – Reasoning 55

Prepare for RBI, SEBI & NABARD, UPSC & UGC NET Exam [email protected]; +91 9999466225

November 170 October +

November + December = 50

0

+ 220 + 500 = 1300

December

(500 – 320 = 180)

Using options given to us we will see that Option A) March 2017 = 160 February 2017 = 100 Increase over previous method = 60/ 100 = 60% Option B) October 2017 = 150 September 2017 = 70 Increase over previous month = 80/ 70 = 114.28% Option C) October of 2016 = 100 September 2016 = 55 Increase over previous period = (45/ 55)* 100 = 88.1% Option D) March of 2016 = 100 February of 2016 = 60 Increase over previous month = 40/60 = 66.66% I.5-7) The table below compares five different countries on various aspects.

Country

Population (in

lakh)

Literacy Rate

Number of

women per 1000 men

Percentage of

rural popula

tion

A 1321 70% 970 45% B 1501 71% 951 62% C 540 85% 1021 51% D 341 90% 992 39% E 832 80% 989 42%

5. What is the approximate number of women in the country which has the second highest number of men among the given countries?

a) 620 Lakh b) 660 Lakh c) 670 Lakh d) 605 Lakh

Ans: b Explanation – Second highest number of men is in country A, as it has second highest population and the number of women per 1000 men is nearly same in all countries Number of women in A = (970/1970)*1321 = less than (½) * 1321 = 650 So, it should be less than half and the final answer should be near 660.

6. Which country has the least number of literate women among the countries listed?

a) D b) C c) E d) Cannot be determined

Ans: a Explanation – Even if all the women in country D are literate, there will be only around 341*1/2 = 170 lakh illiterate women in D (as number of men and women are same) Even if all the men in any country (say C) are literate, out of a total of 540*0.85 = 460 lakh literates, only about 270 lakh (half of total population) can be men and remaining 190 lakh will have to be literate women which is DEFINITELY higher than the number of literate women in D; thus D has least number of literate women.

7. Women form approximately what percentage of the total population of the five countries?

Page 51: Question Paper 2 RBI Grade B 2019 Phase 1 General ... · 4. Who is the brand ambassador of MasterCard? Ans. MS Dhoni [Covered in the Course? – Yes in October Spotlight] 5. Where

RBI Grade B 2019 Phase 1 – Reasoning 56

Prepare for RBI, SEBI & NABARD, UPSC & UGC NET Exam [email protected]; +91 9999466225

a) 45% b) 47% c) 49% d) 51%

Ans: c Explanation – The percentage of women in different countries are 49.2, 48.7, 50.5, 49.8, 49.7 i.e. approximately 49% I.8-11) Answer the questions based on the following information

Year HYDEL THERMAL

NUCLEAR

1960-61 7.8 9.0 - 1970-71 25.2 28.3 2.4 1975-76 25.3 43.3 2.6 1980-81 46.5 61.3 3.0 1985-86 51.0 114.3 5.0 1990-91 54.2 153.8 7.0

Total Power Generated = Hydel power + Thermal Power + Nuclear Power

8. Assuming that the total power generated increased uniformly from 1960-61 to 1970-71, what is the percentage increase in total power generated from 1965-66 to 1975-76?

a) 53% b) 300% c) 89% d) 96%

Ans: d Explanation – Total power generation in (1960-61) = 16.8 Total power generation in (1970-71) = 55.9 Since 1965-66 is exactly between 1960-61 and 1970-71 and the power generated increased uniformly, the power generated in 1965-66 = {16.8+ 55.9}/ 2 = 36.35

Power generated in 1975-76 = 25.3+ 43.3+ 2.6= 71.2 Percentage increase = (34.85/ 36.35)*100 = 96%

9. Which of the following statements is/are true? I. The percentage growth in Hydel Power

generated from 1985-86 to 1990-91 is greater than that of total power generated during the same period.

II. The growth rate in Thermal power generated from 1980-81 to 1985-86 is greater than that of Nuclear power generated during the same period.

III. The growth rate of total power generated from 1975-76 to 1980-81 is more than twice of the same during the period from 1985-86 to 1990-91.

a) Only statement I b) Only statement II c) Only statement III d) Only statement II and III

Ans: d Explanation – we see form the data that only statement II is true. Growth rate in nuclear power from 1980-81 to 1985-86 is 2/3 = 66.67% Also, growth rate in Thermal power from 1980-81 to 1985-86 is 53/61 = 87% So also, growth rate of total power generated from 75-76 to 80-81 is about 55% and that from 85-86 to 90-91 is about 26%. Statement III is also true. Only statement II and III are true.

10. Which of the following statements is/are not true during 1975-76 to 1985-86?

I. The total power generation grew by less than 100%

Page 52: Question Paper 2 RBI Grade B 2019 Phase 1 General ... · 4. Who is the brand ambassador of MasterCard? Ans. MS Dhoni [Covered in the Course? – Yes in October Spotlight] 5. Where

RBI Grade B 2019 Phase 1 – Reasoning 57

Prepare for RBI, SEBI & NABARD, UPSC & UGC NET Exam [email protected]; +91 9999466225

II. The combined power generated from Hydel and Thermal sources grew by a higher percentage than the combined power generated from Thermal and Nuclear sources.

III. In both 1975-76 and 1985-86 the Nuclear power generated was less than 4% of the combined power generated from Thermal and Hydel sources of power.

a) Only statement I and II b) Only statements I and III c) Only statement II d) All three statements

Ans: a

11. Which of the following is TRUE? a) The growth rate of Thermal power generated

from 1975-76 to 1980-81 was not greater than 20%

b) The growth rate of Nuclear Power generated in each five-year period after 1970-71 was greater than 10%

c) The Nuclear power generated in any given year after 1970-71 (where data is available) was never greater than 5% of total power generated during that year.

d) Thermal power generated in 1990-91 formed less than 60% of the total power generated that year. Ans: c I.12) In a school, there are seven classrooms, one each for classes one through eight. These classrooms were built around a circular garden such that classes I to VII are accommodated in that order. The following table gives the sum of the number of students in any group of three consecutively suited classes.

Classes Total Students I, II, III 300

II, III, IV 280 III, IV, V 240 IV, V, VI 195 V, VI, VII 215 VI, VII, I 235 VII, I, II 260

The percentage of girls in different classes are 30%, 40%, 44%, 45%, 50%, 55% and 60% not necessarily in any order.

12. If it is known that X classes have an equal number of girls then what is the maximum possible value of X?

a) 2 b) 3 c) 4 d) 5

Ans: b Explanation – The possible number of girls for I, VII- 24,32,36,40,44,48 II- 30,40,44,45,50,55,60 III- 36,48,54,60,66,72 IV, V- 18,24,27,30,33,36 VI- 30,33,45 Here, only 30 (or 36) can occur for 3 times II- 30% of 100 = 30 IV – 50% of 60 = 30 V – 40% of 75= 30 I.13-16) The question is followed by two statements, A and B, giving certain data. You have to mark the correct answer depending on the sufficiency of data given in statement to answer the questions. Mark (1): If the question can be answered using one of the statements alone but cannot be answered by using other statements alone.

Page 53: Question Paper 2 RBI Grade B 2019 Phase 1 General ... · 4. Who is the brand ambassador of MasterCard? Ans. MS Dhoni [Covered in the Course? – Yes in October Spotlight] 5. Where

RBI Grade B 2019 Phase 1 – Reasoning 58

Prepare for RBI, SEBI & NABARD, UPSC & UGC NET Exam [email protected]; +91 9999466225

Mark (2): If the question can be answered by using either statement alone Mark (3): If question can be answered by using both statements together but cannot be answered by using either statement alone. Mark (4): If question cannot be answered by using both statements together

13. If the HCF of two numbers is 24, then what is the ratio of larger number to the smaller number?

I. The LCM of two numbers is 120 II. The sum of the two numbers is 144

a) Mark 1 b) Mark 2 c) Mark 3 d) Mark 4

Ans: b Explanation – Given HCF of the two numbers is 24. Let the numbers be 24x and 24y. Using statement, A alone, LCM of two numbers is 120. LCM of 24x and 24y is 120 (24x) *(24y) = 120 *24 XY= 5 X and y can be only 1 and 5 in any order. Required ratio = 5 Using statement B alone, sum of two numbers is 144. 24x+ 24y = 144 X+y = 6 X, Y cannot 2,4 and 3,3 as the HCF is 24. They must be 1 and 5 in any order. Required ratio = 5

14. The speeds of train A and train B are 72 kmph and 18 kmph respectively. What is the length of the train A?

I. Train A crosses train B in 50 seconds II. Train A crosses a car which is moving at 18

kmph in 35 seconds. a) Mark 1 b) Mark 2 c) Mark 3 d) Mark 4

Ans: d Explanation – Let the lengths of train A and train B be x meters and y meters respectively. Case 1: The two trains are moving in the same direction = x+y = (72-18) * (5/18) * (50) = 750 m Case 2: The two trains are moving in opposite direction (x+y) = (72+18) *5/18* 50 = 1250 m We do not know the value of y, so we can’t find X. So, statement I alone is not sufficient. From Statement II Case I: Train and Car are moving in the same directions Then X = (72-18) *(5/18) *(35) = 525 m Case II: Train and Car are moving in opposite direction Then (72+ 18) *(5/18)*(35) = 875 m So, second alone is NOT sufficient Even by using both the statements together, we cannot define the value of X.

15. The cost price of three articles P, Q and R are in

the ratio 3:7:5 and their selling price are in the ratio of 1:4:2 respectively. What is the cost price of article R?

I. The difference between the cost price and selling price of three articles is same and is equal to Rs 100.

II. Profit is made on only one of the three articles. *[a] Mark 1 [b] Mark 2 [c] Mark 3

Page 54: Question Paper 2 RBI Grade B 2019 Phase 1 General ... · 4. Who is the brand ambassador of MasterCard? Ans. MS Dhoni [Covered in the Course? – Yes in October Spotlight] 5. Where

RBI Grade B 2019 Phase 1 – Reasoning 59

Prepare for RBI, SEBI & NABARD, UPSC & UGC NET Exam [email protected]; +91 9999466225

[d] Mark 4 Ans: a Explanation - Case 1 (If loss is made only on R or loss is made only on P and Q)

Case 2(If loss is made only on Q or loss is made only on P and R then)

Case 3 (if loss is made only on P or loss is made only on Q and R then)

3x-y = 7x-4y and 7x-4y = 2y-5x x/y = ¾ and x/y =3

3x-y = 4y-7x=5x-2y x/y = ½ and x/y =½

Y-3x=7x-4y= 5x-2y 5y=10x and 2x=2y x/y =½ and x/y =1

As values doesn’t match. It is invalid.

Values match, case is valid

Values doesn’t match, case does not match

Difference of C.P and S.P of P = |3x-y| X=k, Y=2k D= k =100 Cost price = 5k =500 Only statement 1 is sufficient.

16. In a B-school, 30% of the students in a batch were male. If 10% of the females students have work experience, what percent males have work experience?

I. 20% of all the students in the batch have work experience

II. The number of males having work experience is 25% more than the number of females having work experience.

a) Mark 1 b) Mark 2 c) Mark 3 d) Mark 4

Ans: b

Explanation – Let the number of students in the batch be =x Then number of male students are = 0.3x and 0 % of females have work experience. = 0.07 x females have work experience Using I alone is sufficient Using II alone, the number of males who do not work is 0.13x Hence, alone II is sufficient. I.17) Observation over a period of 1 year has established the following pattern of sales in Fencer in Bharani Colony?

Bill Amount Percentage of Customers

Rs. 0 to Rs. 100 60% Rs.100 to Rs.200 10% Rs.200 to Rs.300 10%

More than Rs.300 20%

17. What is the probability that of any four customers, exactly two have a bill between Rs. 200 and Rs. 300?

a) 0.81% b) 2.435% c) 4.86 d) 8.1%

Ans: c Explanation – For any customer, the probability that the bill is between Rs 200 to Rs.300 is 10%, while the probability that it is something else is 90%. The required probability is 4C2 (0.1)2(0.9)2 6(0.01)(0.81) = 4.86%

18. In a certain class, the number of students who do not study at home or do not attend classes is one third more than that of those who either study at home or attend classes. Further, the number of students who do not study at home

Page 55: Question Paper 2 RBI Grade B 2019 Phase 1 General ... · 4. Who is the brand ambassador of MasterCard? Ans. MS Dhoni [Covered in the Course? – Yes in October Spotlight] 5. Where

RBI Grade B 2019 Phase 1 – Reasoning 60

Prepare for RBI, SEBI & NABARD, UPSC & UGC NET Exam [email protected]; +91 9999466225

but attend classes is two fifth more than those who study at home but do not attend classes; while the number of students who study at home as well as attend classes is half that of those who neither study at home nor attend classes. If the total number of students in the class is 150, then find the number of students who study at home as well as attend classes.

a) 35 b) 30 c) 25 d) 45

Ans: b Explanation -

Number who don’t

attend classes

Number who attend

classes

Number who don’t study at

home

A B

Number who study at

home

D C

Given (a+b+d) = (1+1/3)(C+D+B) (a+b+d) = 4/3 (C+ D+ B) -------> 1 Further b = (1+ 2/5) D and C = A/2 A = 2c, b = (7/5) D -------> 2 Substituting 2 in 1 2 C + 7/5 D + D = 4/3 (C+ D + 7/5 D) 2 C + (12/5) D = (4/3) C + (16/5) D 2/3 c = 4/5 d ----------> 3 D = (5/6) C B = 7/5 (5/6) C A = 2c A + B + C + D = 5c = 150 C = 30

19. If a and c are positive, and the roots of the quadratic equation (a^2+b^2) x^2 + 2(ab+bc)x + (b^2+c^2) = 0 are real, find the minimum possible value of the quadratic expression ax^2 + bx+c

a) -3c/ 4 b) -c/2 c) c/2 d) 3c/4

Ans: d Explanations – Discriminant of the quadratic equation = {2 (ba+bc)}^2 – 4(a^2 +b^2) (b^2+c^2) = -4(b^2 – ac^2) As the roots are real the discriminant ≥ 0. Hence, b^2- ac must be equal to 0. = b^2= ac The minimum value of the quadratic expression ax^2 +bx+c = {4ac-b^2)/4a = 3c/4 I.20-23) There are only four brands of entry level smartphones called Azra, Bysi, Cxqi, and Dipq in a country. Details about their market share, unit selling price, and profitability (defined as the profit as a percentage of the revenue) for the year 2016 are given in the table below:

In 2017, sales volume of entry level smartphones grew by 40% as compared to that in 2016. Cxqi offered a 40% discount on its unit selling price in 2017, which resulted in a 15% increase in its market share. Each of the other three brands lost 5% market share. However, the profitability of Cxqi came down to half of its value in 2016. The unit selling prices of the

Page 56: Question Paper 2 RBI Grade B 2019 Phase 1 General ... · 4. Who is the brand ambassador of MasterCard? Ans. MS Dhoni [Covered in the Course? – Yes in October Spotlight] 5. Where

RBI Grade B 2019 Phase 1 – Reasoning 61

Prepare for RBI, SEBI & NABARD, UPSC & UGC NET Exam [email protected]; +91 9999466225

other three brands and their profitability values remained the same in 2017 as they were in 2016.

20. The brand that had the highest revenue in 2016

is: a) Dipq b) Bysi c) Cxqi d) Azra

Ans: d Explanations – For 2016 Let the total market size be 100

Brand

Market

Share

Selling

Price

Profitability

Revenue Profit

Azra 40% 15,000 10% 6,00,00

0

6,00,000 * 0.10 = 60,000

Bysi 25% 20,000 30% 5,00,00

0

5,00,000* 0.30 = 1,50,000

Cxqi 15% 30,000 40% 4,50,00

0

4,50,000 * 0.40 = 1,80,000

Dipq 20% 25,000 30% 5,00,00

0

5,00,000* 0.3 =

1,50,000 For 2017

Person

Market Share

Selling Price

Profitability

Revenue Profit

Azra 49%

15,000 10% 7,35,00

0

7,35,000 * 0.1 = 73,500

Bysi 28%

20, 000 30% 5,60,00

0

5,60,000 * 0.3 =

1,68,000

Cxqi 42%

18,000 20% 7,56,00

0

7,56,000 * 0.2 = 1,51,200

Dipq 21%

25,000 30% 5,25,00

0

5,25,000 * 0.3 = 1,57,500

As the sales volume increased by 40%, so total sales volume is 140. Market Share of A = 40 – 5 = 35% and 40% rise in this 35% equals to 49% as increase. Market Share of B = 25-5 = 20% and 40% rise in this 20% equals to 8% as increase Market Share of C = 15+5 = 30% and 40% rise in this 30% equals to 12% increase Market Share of D = 20 –5 = 15% and 40% rise in this 15% equals to 6% increase. The highest revenue for 2016 is – Rs 6,00,000 by Azra

21. The brand that had the highest profit in 2016 is:

a) Azra b) Bysi c) Cxqi d) Dipq

Ans: c Explanation - For 2016 Let the total market size be 100

Page 57: Question Paper 2 RBI Grade B 2019 Phase 1 General ... · 4. Who is the brand ambassador of MasterCard? Ans. MS Dhoni [Covered in the Course? – Yes in October Spotlight] 5. Where

RBI Grade B 2019 Phase 1 – Reasoning 62

Prepare for RBI, SEBI & NABARD, UPSC & UGC NET Exam [email protected]; +91 9999466225

Person

Market

Share

Selling

Price

Profitability

Revenue Profit

Azra 40% 15,000 10% 6,00,00

0

6,00,000 *

0.10 = 60,000

Bysi 25% 20,000 30% 5,00,00

0

5,00,000 *

0.30 = 1,50,00

0

Cxqi 15% 30,000 40% 4,50,00

0

4,50,000 *

0.40 = 1,80,00

0

Dipq* 20% 25,000 30% 5,00,00

0

5,00,000 * 0.3

= 1,50,00

0 For 2017

Person

Market

Share

Selling

Price

Profitability

Revenue Profit

Azra 49% 15,000 10% 7,35,00

0

7,35,000* 0.1 = 73,500

Bysi 28% 20, 000 30% 5,60,00

0

5,60,000* 0.3 =

1,68,000

Cxqi 42% 18,000 20% 7,56,00

0

7,56,000 * 0.2=

1,51,200

Dipq 21% 25,000 30% 5,25,00

0

5,25,000* 0.3

=1,57,500

As the sales volume increased by 40%, so total sales volume is 140.

Market Share of A = 40 – 5 = 35% and 40% rise in this 35% equals to 49% as increase. Market Share of B = 25-5 = 20% and 40% rise in this 20% equals to 8% as increase Market Share of C = 15+5 = 30% and 40% rise in this 30% equals to 12% increase Market Share of D = 20 –5 = 15% and 40% rise in this 15% equals to 6% increase. The brand that had highest profit in 2016 was – Cxqi – 1,80,000

22. The brand that had the highest profit in 2017

is? a) Dipq b) Bysi c) Cxqi d) Azra

Ans: b Explanation - For 2016 Let the total market size be 100

Person

Market

Share

Selling

Price

Profitability

Revenue Profit

Azra 40% 15,000 10% 6,00,00

0

6,00,000 * 0.10 = 60,000

Bysi 25% 20,000 30% 5,00,00

0

5,00,000 * 0.30 = 1,50,000

Cxqi 15% 30,000 40% 4,50,00

0

4,50,000 * 0.40 = 1,80,000

Dipq 20% 25,000 30% 5,00,00

0

5,00,000* 0.3 =

1,50,000

Page 58: Question Paper 2 RBI Grade B 2019 Phase 1 General ... · 4. Who is the brand ambassador of MasterCard? Ans. MS Dhoni [Covered in the Course? – Yes in October Spotlight] 5. Where

RBI Grade B 2019 Phase 1 – Reasoning 63

Prepare for RBI, SEBI & NABARD, UPSC & UGC NET Exam [email protected]; +91 9999466225

For 2017

Person

Market

Share

Selling

Price

Profitability

Revenue Profit

Azra 49% 15,000 10% 7,35,00

0

7,35,000 * 0.1

= 73,500

Bysi 28% 20, 000 30% 5,60,00

0

5,60,000 * 0.3

= 1,68,00

0

Cxqi 42% 18,000 20% 7,56,00

0

7,56,000 * 0.2

= 1,51,20

0

Dipq 21% 25,000 30% 5,25,00

0

5,25,000 * 0.3

= 1,57,50

0 As the sales volume increased by 40%, so total sales volume is 140. Market Share of A = 40 – 5 = 35% and 40% rise in this 35% equals to 49% as increase. Market Share of B = 25-5 = 20% and 40% rise in this 20% equals to 8% as increase Market Share of C = 15+5 = 30% and 40% rise in this 30% equals to 12% increase Market Share of D = 20 –5 = 15% and 40% rise in this 15% equals to 6% increase. Highest profit in 2017 was 1,68,000 BY Bysi.

23. The complete list of brands whose profits went up in 2017 from 2016 is:

a) Azra, Bysi, Cxqi b) Bysi, Cxqi, Dipq c) Cxqi, Azra, Dipq d) Azra, Bysi, Dipq

Ans: d

Explanation: For 2016 Let the total market size be 100

Person

Market

Share

Selling

Price

Profitability

Revenue Profit

Azra 40% 15,000 10% 6,00,00

0

6,00,000 *

0.10 = 60,000

Bysi 25% 20,000 30% 5,00,00

0

5,00,000 *

0.30 = 1,50,00

0

Cxqi 15% 30,000 40% 4,50,00

0

4,50,000 *

0.40 = 1,80,00

0

Dipq 20% 25,000 30% 5,00,00

0

5,00,000 * 0.3

= 1,50,00

0 For 2017

Person

Market

Share

Selling

Price

Profitability

Revenue Profit

Azra 49% 15,000 10% 7,35,00

0

7,35,000 * 0.1

= 73,500

Bysi 28% 20, 000 30% 5,60,00

0

5,60,000 * 0.3

= 1,68,00

0

Cxqi 42% 18,000 20% 7,56,00

0

7,56,000 * 0.2

= 1,51,20

0

Page 59: Question Paper 2 RBI Grade B 2019 Phase 1 General ... · 4. Who is the brand ambassador of MasterCard? Ans. MS Dhoni [Covered in the Course? – Yes in October Spotlight] 5. Where

RBI Grade B 2019 Phase 1 – Reasoning 64

Prepare for RBI, SEBI & NABARD, UPSC & UGC NET Exam [email protected]; +91 9999466225

Dipq 21% 25,000 30% 5,25,00

0

5,25,000 * 0.3

= 1,57,50

0

As the sales volume increased by 40%, so total sales volume is 140. Market Share of A = 40 – 5 = 35% and 40% rise in this 35% equals to 49% as increase. Market Share of B = 25-5 = 20% and 40% rise in this 20% equals to 8% as increase Market Share of C = 15+5 = 30% and 40% rise in this 30% equals to 12% increase Market Share of D = 20 –5 = 15% and 40% rise in this 15% equals to 6% increase. The brands whose profit went up were: Azra, Bysi and Dipq I.24-27) Answer the questions based on the information given below. X@Y – Gives the positive difference of X and Y X$Y- Gives the sum of squares of X and Y X#Y – Gives the positive difference of the squares of X and Y X& Y – Gives the product of X and Y Also, X, Y belong to R. The other standard algebraic operations are unchanged.

24. Given that x@y=x-y, then find (x$y)+(x#y) a) 2x^2 b) 2(y)^2 c) 2(x^(2)+ y^(2)) d) Cannot be determined

Ans: d Explanation – Given X@Y = X-Y The positive difference of X and Y is X-Y= X>Y But still we cannot conclude anything about the positive difference of the squares of X and Y, since say X =1 and Y=-3

X@Y= X-Y and X#Y = (Y)^2 -(X)^2 but if x=3 and y=1 Then X#Y= (X)^2- (Y)^2, we cannot find the expression.

25. The expression {(x#y) + (x@y)}^2-2(x&y) will be equal to

a) x#y b) X$Y c) (X#Y)(X@Y) d) Cannot be determined

Ans: b Explanation – Given {(X^2 – y^2)/(X-Y)}^2 – 2XY Where a-b = positive difference of a and b {(x^2- y^2)/(X-Y)} ^2 –2xy = X$Y

26. Given that x@y=x-y, then find (x$y)+(x#y) a) 2x^2 b) 2(y)^2 c) 2(x^(2)+ y^(2)) d) Cannot be determined

Ans: d Explanation – Given X@Y = X-Y The positive difference of X and Y is X-Y= X>Y But still we cannot conclude anything about the positive difference of the squares of X and Y, since say X =1 and Y=-3 X@Y= X-Y and X#Y = (Y)^2 -(X)^2 but if x=3 and y=1 Then X#Y= (X)^2- (Y)^2, we cannot find the expression.

27. The expression {(x#y) +(x@y)}^2-2(x&y) will be equal to

a) x#y b) X$Y c) (X#Y)(X@Y) d) Cannot be determined

Ans: b

Page 60: Question Paper 2 RBI Grade B 2019 Phase 1 General ... · 4. Who is the brand ambassador of MasterCard? Ans. MS Dhoni [Covered in the Course? – Yes in October Spotlight] 5. Where

RBI Grade B 2019 Phase 1 – Reasoning 65

Prepare for RBI, SEBI & NABARD, UPSC & UGC NET Exam [email protected]; +91 9999466225

Explanation – Given {(X^2 – y^2)/(X-Y)}^2 – 2XY Where a-b = positive difference of a and b {(x^2- y^2)/(X-Y)]}^2 –2xy = X$Y

28. The 4800th term of the sequence

1,3,3,3,5,5,5,5,5,7,7,7,7,7,7,7,9,….............will be?

a) 138 b) 139 c) 140 d) 137

Ans: b Explanation – The sequence shows that last digit of every number falls at an odd place and the position is at the square of number. So, the number 4800 lies in square of 69 and 70 Now to find the exact position we use the formulae of 2(70) -1= 139

29. A total of 768 balls are arranged in a pile in 9

layers. The nth layer from the top, where 3≤ n ≤ 9 has as many balls as the sum of the number of balls in all layers above it. Find the total number of balls in the 6th and 7th layers from the top put together.

a) 72 b) 144 c) 288 d) 216

Ans: b Explanation – Let the number of balls in top 5 layers be total of X. Now, in sixth layer, the balls are X. So, in 7 the layer balls are equal to total balls in first six layers, that is 2x. In layer 8 the balls are equal to X+ X+ 2X= 4X In layer 9, the balls are equal to X+X+2X+4X = 8X.

So total number of balls are 8x+ 4x+ 2x+ x + x= 16x = 768 So, x=48. The total number of balls in 6th and 7th layer is 3x= 3(48) = 144

30. In a colony, K friends R1, R2, R3 …...... Rk stand clockwise in a circle with R2 to the left R1, R3 left to the R2, R4 to the left to the R3 and so on till Rk. R1 is to the left of Rk. Each of the friends either always lies or always speaks the truth. For each j=1 to k, Rj speaks about the person to the left. If j is a perfect square Rj says that person to her left is lying and for all other j, Rj says that the person to her left is speaking the truth. Which of the following ranges contains a possible value of k?

a) 1050<K< 1125 b) 825 < K < 899 c) 1240< K < 1320 d) More than one of the above

Ans: c Explanation – Suppose R9 speaks the truth = R1 is lying R2 speaks the truth = R3 and R4 speak the truth R4 is lying = R6, R7, R8, R9 are lying. But this is a contradiction to the initial assumption that R9 speaks the truth. Of the given ranges, only option, 1240<k< 1320 (32) ^2 = 1024 (34) ^2 = 1156, are not present in given ranges